Вы находитесь на странице: 1из 81

1

INDIAN BANKING SYSTEM:

Nationalisation of Indian Banks and up


to 1991 prior to Indian banking sector
reforms.

New phase of Indian Banking System


with the advent of Indian Financial &
Banking Sector reforms after 1991.

A DETAILED STUDY
BANKING SYSTEM IN
INDIA
Banking system occupies an important
place in a nations economy. A banking
institution is indispensable in a modern society.
It plays a pivotal role in the economic
development of a country and forms the core of
the money market in an advanced country.
For the past three decades Indias
banking system has several outstanding
achievements to its credit. The most striking is
its extensive reach. It is no longer confined to
only metropolitans or cosmopolitans in India. In
fact, Indian banking system has reached even to
the remote corners of the country. This is one of
the main reasons of Indias growth process.
The governments regular policy for
Indian bank since 1969 has paid rich dividends
with the nationalization of 14 major private
banks of India.
Not long ago, an account holder had to
wait for hours at the bank counters for getting a
draft or for withdrawing his own money. Today,
he has a choice. Gone are days when the most
efficient bank transferred money from one
branch to other in two days. Now it is as simple
as instant messaging or dial a pizza. Money has
become the order of the day.
The first bank in India, though
conservative, was established in 1786. From
1786 till today, the journey of Indian Banking
System. Can be segregated into three distinct
phases. They are as mentioned below:

Early phase from 1786 to 1969 of Indian


Banks

EARLY PHASE OR PHASE I


The General Bank of India was set up in
the year 1786, next came Bank of Hindustan and
Bengal Bank. The East India Company
established Bank of Bengal (1809), Bank of
Bombay (1840) and Bank of Madras (1843) as
independent units and called it Presidency
Banks. These three banks were amalgamated in
1920 and Imperial Bank of India was established
which started as private shareholders banks,
mostly Europeans shareholders.
In 1865 Allahabad Bank was established
and first time exclusively by Indians, Punjab
National Bank Ltd. was set up in 1894 with
headquarters at Lahore. Between 1906 and 1913,
Bank of India, Central Bank of India, Bank of
Baroda, Canara Bank, Indian Bank, and Bank of
Mysore were set up. Reserve Bank of India
came in 1935.
During the First World War (1914-1918)
through the end of the Second World War (19391945), and two years thereafter until the
independence of India were challenging for
Indian banking. The years of the First World
War were turbulent, and it took its toll with
banks simply collapsing despite the Indian
economy gaining indirect boost due to warrelated economic activities. At least 94 banks in

India failed between 1913 and 1918 as indicted


in the following table:

2
Authorised Capital

Paid-up Capital

(Rs.Lakhs)

(Rs.Lakhs)

12

274

35

1914

42

710

109

1915

11

56

1916

13

231

1917

76

25

1918

209

Years

Number of banks that failed

1913

To streamline the functioning and activities of


commercial banks. The Government of India
came up with the Banking Companies Act, 1949
which was later changed to Banking Regulation
Act 1949 as per amending Act of 1965 (Act No.
23 of 1965). Reserve Bank of Indias was vested
with extensive powers for the supervision of
banking in India as the Central Banking
Authority.
During those days public has lesser
confidence in the banks. As an aftermath deposit
mobilization was slow Abreast of it the savings
bank facility provided by the Postal department
was comparatively safer. Moreover, funds were
largely given to traders.
NATIONALISATION
PHASE II

OF

BANKS

Resolution adopted by the government in 1948


envisaged a mixed economy. This resulted into
greater involvement of the state in different
segments of the economy including banking and
finance. The major steps to regulate banking in
cluded:

The Reserve Bank of India, Indias


central
banking
authority,
was
nationalized on January 1, 1949 under
the terms of the Reserve Bank of India
(Transfer to Public Ownership) Act,
1948
(RBI,
2005b).
[Reference
www.rbi.org.in]

In 1949, the Banking Regulation Act


was enacted which empowered the
Reserve Bank of India [RBI] to
regulate, control, and inspect the banks
in India.

The Banking Regulation Act also


provided that no new bank or branch of
an existing bank could be opened
without a license from the RBI, and no
two banks could have common
directors.

OR

Government took major steps in this


Indian
Banking
Sector
Reform
after
independence. The partition of India in 1947
adversely impacted the economies of Punjab and
West Bengal, paralyzing banking activities for
months. Indias independence marked the end of
a regime of the Laissez-faire for the Indian
banking. The Government of India initiated
measures to play an active role in the economic
life of the nation, and the industrial Policy

In 1955, it nationalized Imperial Bank of


India with extensive banking facilities on a large

3
scale specially in rural and semi-urban areas. It
formed State Bank of India to act as the
principal agent of RBI and to handle banking
transactions of the Union and State
Governments all over the country.
Despite the provisions, control and
regulations of Reserve Bank of India, banks in
India except the State Bank of India or SBI,
continued to be owned and operated by private
persons. By the 1960s, the Indian banking
industry had become an important tool to
facilitate the development of the Indian
economy. At the same time,. It had emerged as a
large employer, and a debate had ensued about
the nationalization of the banking industry.
Indira Gandhi., then Prime Minister of India,
expressed the intention of the Government of
India in the annual conference of the All India
Congress Meeting in a paper entitled Stray
thoughts on Bank Nationalisation. The meeting
received the paper with enthusiasm.
Thereafter, her move was swift and
sudden. The Government of India issued an
ordinance and nationalized the 14 largest
commercial banks with effect from the midnight
of July 19, 1969. Jayaprakash Narayan, national
leader cal sagacity. Within two weeks of the
issue of the ordinance, the Parliament passed the
Banking Companies (Acquisition and Transfer
of Undertaking) Bill, and it received the
presidential approval on 9 August 1969.
A second dose of nationalization of 6
more commercial banks followed in 1980. The
stated reason for the nationalization was to give
the government more control of credit delivery.
With the second dose of nationalization, the
Government of India controlled around 91% of
the banking business of India, Later on, in the
year 1993, the government merged new Bank of
India with Punjab National Bank. It was the only
merger between nationalized banks and resulted
in the reduction of the number of nationalized

banks from 20 to 19. After this, until the 1990s,


the nationalized banks grew at a pace of around
4%, closer to the average growth rate of the
Indian economy.
The following are the steps taken by the
Government of India to regulate Banking
Institutions in the Country:

1949: Enactment of Banking Regulation


Act.

1955: Nationalisation State Bank of


India.

1959:
Nationalisation
subsidiaries.

1961: Insurance cover extended to


deposits.

1969 : Nationalisation of 14 major


banks

1971 : Creation of credit guarantee


corporation.

1975 : Creation of regional rural banks.

1980 : Nationalisation of seven banks


with deposits over Rs. 200 crore.

of

SBI

After the nationalization of banks, the


branches of the public sector bank of India rose
to approximately 800% in deposits and advances
took a huge jump by 11,000%.
Banking in the sunshine of government
ownership gave the public implicit faith and
immense confidence about the sustainability of
these institutions.
FINANCIAL OR BANKING
REFORMS OR PHASE III

SECTOR

This phase has introduced many more


products and facilities in the banking sector in

4
its reforms measure. In 1991, under the
chairmanship of M Narasimham, a committee
was set up by his name which worked for the
liberalization of banking practices. In the early
1990s, the then Narsimha Rao government
embarked on a policy of liberalization, licensing
a small number of private banks. These came to
be known as New Generation tech/savvy banks,
and included Global Trust Bank (the first of such
new generation banks to be set up), which later
amalgamated with Oriental Bank of Commerce
Axis Bank (earlier as UTI Bank), ICICI Bank
and HDFC Bank. This move, along with the
rapid growth in which has seen rapid growth
with strong contribution from all the three
sectors of banks, namely government banks
private banks and foreign banks.
The next stage for the Indian banking
has been set up with the proposed relaxation in
the norms for Foreign Direct Investment, where
all Foreign Investors in banks may be given
voting rights which could exceed the present cap
of 10%, at present it has gone up to 74% with
some restrictions.
The new policy shook the Banking
sector in India completely. Bankers, till this
time, were used to the 4-6-4 method (Borrow at
4%; Lend at 6%; Go home at 4) of functioning.
The new wave ushered in a modern outlook and
tech-savvy methods of working for traditional
banks. All this led to the retail boom in India.
People not just demanded more from their banks
but also received more.
Currently (2007), banking in India is
generally fairly mature in terms of supply.
Product range and reach-even though reach in
rural India still remains a challenge for the
private sector and foreign banks. In terms of
quality of assets and capital adequacy, Indian
banks are considered to have clean, strong and
transparent balance sheets relative to other banks
in comparable economies in its region. The

Reserve Bank of India is an autonomous body,


with minimal pressure from the government.
The stated policy of the Bank on the Indian
Rupee is to manage volatility but without any
fixed exchange rate-and this has mostly been
true.
With the growth in the Indian economy
expected to be strong for quite some timeespecially in its services sector-the demand for
banking services, especially retail banking
mortgages and investment services are expected
to be strong. One may also expect M&As,
takeovers, and asset sales.
In March 2006, the Reserve Bank of
India allowed Warburg Pincus to increase its
stake in Kotak Mahindra Bank (a private sector
bank) to 10%. This is the first time an investor
has been allowed to hold more than 5% in a
private sector bank since the RBI announced
norms in 2005 that any stake exceeding 5% in
the private sector banks would need to be vetted
by them.
The country is flooded with foreign
banks and their ATM stations, Efforts are being
put to give a satisfactory service to customers.
Phone banking and net banking are introduced.
The entire system became more convenient and
swift. Time is given more importance than
money.
The financial system of India has shown
a great deal of resilience. It is sheltered from any
crisis triggered by any external macroeconomics
shock as other East Asian countries suffered.
This is all due to a flexible exchange rate
regime, the foreign reserves are high , the capital
account is not yet fully convertible, and banks
their customers have limited foreign exchange
exposure.
NATIONALISATION OF BANKS IN INDIA

5
The nationalization of banks in India
took place in 1969 by Mrs. Indira Gandhi, the
then Prime Minister. It nationalized 14 banks

then. These banks were mostly owned by


businessmen and even managed by them.

Central Bank of India

Indian Overseas Bank

Bank of Maharashtra

Bank of Baroda

Dena Bank

Union Bank

Punjab National Bank

Allahabad Bank

Syndicate Bank

United Bank of India

Canara Bank

UCO Bank

Indian Bank

Bank of India

Before the steps on nationalization of


Indian banks, only State Bank of India (SBI)
was nationalized. It took place in July 1955
under the SBI Act of 1955. Nationalisation of
Seven State Banks of India (formed subsidiary)
took place on 19th July, 1959.
The State Bank of India is Indias largest
commercial bank and is ranked one of the top
five banks worldwide. It serves 90 million

customers through a network of 9,000 branches


and it offers-either directly or through
subsidiaries-a wide range of banking services.
The second phase of nationalisation of
Indian banks took place in the year 1980. Seven
more banks were nationalized with deposits over
Rs.200 crores. Till 2006, approximately 80% of
the banking segments in India were under
Government ownership.

SCHEDULED COMMERCIAL
BANKS IN INDIA

The commercial banking structure in India


consists of:

Scheduled Commercial Banks in India

Unscheduled Banks in India

Scheduled Banks in India constitute


those banks which have been included in the
Second Schedule of Reserve bank of India (RBI)
Act, 1934. RBI in turn includes only those banks
in this schedule which satisfy the criteria laid
down vide section 42 (6) (a) of the Act.

As on 30th June, 1999, there were 300


scheduled banks in India having a total network
of 64,918 branches. The scheduled commercial
banks in India comprise of State bank of India
and its associates (8), nationalized banks (19),
foreign banks (45), private sector banks (32), cooperative banks and regional rural banks.
Scheduled banks in India means the
State Bank of India constituted under the State
bank of India Act, 1955 (23 of 1955), a
subsidiary bank as defined in the State bank of
India (Subsidiary Banks) Act, 1959)(38 of 1959)
a corresponding new bank constituted under

6
section 3 of the Banking Companies)
Acquisition and Transfer of Undertakings) Act,
1970 (5 of 1970), or under section 3 of the
Banking Companies (Acquisition and Transfer
of Undertaking) Act, 1980 (40 of 1980), or any
other bank being a bank included in the Second
Schedule to the Reserve Bank of India Act, 1934
The following are the Scheduled Banks in
India (Public Sector):

(2 of 1934), but does not include a cooperative


bank.
Non- scheduled bank in India means a
banking company as defined in clause (c) of
section 5 of the Banking Regulation Act, 1949
(10 of 1949), which is not a scheduled bank

Central Bank of India

Corporation Bank

Dena Bank

State Bank of India

State Bank of Bikaner and jaipur

State Bank of Hyderabad

Indian Overseas Bank

State bank of Indore

Indian Bank

State Bank of Mysore

Oriental Bank of Commerce

State Bank of Saurashtra

Punjab National Bank

State Bank of Travancore

Andhra Bank

Punjab and Sind Bank

Allahabad Bank

Syndicate Bank

Bank of Baroda

Union Bank of India

Bank of India

United Bank of India

Bank of Maharashtra

UCO Bank

Canara Bank

Vijaya Bank

Centurion Bank Ltd

Bank of Punjab Ltd

IDBI Bank Ltd

Jammu & Kashmir Bank Ltd

The following are the Scheduled Banks in


India (Private Sector):

ING Vysya Bank Ltd

Axis Bank Ltd

Indusind Bank Ltd

The following are the Scheduled Foreign


banks in India:

ICICI Bank Ltd

South Indian Bank

American Express Bank Ltd.

HDFC Bank Ltd

ANZ Gridlays Bank Plc.

Bank of America NT & SA

Deustche Bank A.G.

Bank of Tokyo Ltd

Banquc Nationale de Paris

Hongkong and
Corporation

Standard Chartered Bank.

Barclays Bank Plc

The Chase Manhattan Bank Ltd.

Citi Bank N.C.

Dresdner Bank AG.

Public Sector Banks in India


Among the Public Sector Banks in
India, United Bank of India is one of the 14
major banks which were nationalized on July 19,
1969. Its predecessor, in the Public Sector
Banks, the United Bank of India Ltd., was
formed in 1950 with the amalgamation of four
banks viz. Comilla banking Corporation Ltd.
(1914), Bengal Central bank Ltd. (1918),
Comilla Union Bank Ltd. (1922) and Hooghly
Bank Ltd. (1932).
Oriental bank of Commerce (OBC), a
Government of India Undertaking offers
Domestic, NRI and Commercial banking
services. OBC is implementing a GRAMEEN
PROJECT in Dehradhun District (Uttarkhand)
and
Hanumangarh
District
(Rajasthan)
disbursing small loans. This public Sector Bank
in India has implemented 14 point action plan
for strengthening of credit delivery to women
and has designated 5 branches as specialized
branches for women entrepreneurs.
Private Sector Banks in India
Private banking in India was practised
since the beginning of banking system in India.
The first private bank in India to be set up in
Private Sector banks in India was Indusind
Bank. It is one of the fastest growing Private
Sector Banks in India. IDBI ranks the tenth
largest development bank in the world as Private

Shanghai

Banking

banks in India and has promoted a world class


institutions in India.
The first Private bank in India to receive
an in-principle approval from the Reserve Bank
of India was Housing Development Finance
Corporation Limited, to set up a bank in the
private sector banks in India as part of RBIs
liberalization of the Indian banking Industry. It
was incorporated in August 1994 as HDFC Bank
Limited with registered office in Mumbai and
commenced
operations
as
Scheduled
Commercial bank in January 1995.
,ING Vysya, yet another Private bank of India
was in corporated in the year 1930. Bangalore
has a pride of place for having the first branch
inception in the year 1934. With successive
years of patronage and constantly setting new
standards in banking, ING Vysya Bank has
many credits to its account.
Foreign Banks in India
Foreign banks in India always brought an
explanation about the prompt services to
customers. After the set up foreign banks in
India the hankino sector in India also become
competitive and accurative.
New rules announced by the Reserve
Bank of India for the foreign banks in India in
this budget has put up great hopes among
foreign banks which allows them to grow
unfettered. Now foreign banks in India are
permitted to set up local subsidiaries. The policy

8
conveys that foreign banks in India may not
acquire Indian ones (except for weak banks
identified by the RBI, on its terms) and their
Indian subsidiaries will not be able to open
branches freely.

List of Foreign Banks in India

ABN-AMRO Bank

Deutsche Bank

Abu Dhabi Commercial bank

HSBC

Bank of Ceylon

JPMorgan Chase Bank

BNP Paribas Bank

Standard Chartered Bank

Citi Bank

Scotia Bank

China Trust Commercial Bank

Taib Bank

Industrial and Commercial Bank of


China

By the year 2009, the list of foreign banks


in India is going to become more quantitative as
numbers of foreign banks are still waiting with
baggage to start business in India.
Upcoming Foreign Banks in India
By the year 2009 few more names is going to
be added in the list of foreign banks in India.
This is as an aftermath of the sudden interest
shown by Reserve Bank of India paving road
map for foreign banks in India with greater
freedom. Among them is the worlds best private
bank by Euro Money magazine, Switzerlands
UBS.
The following are the list of foreign banks
going to set up business in India

Royal Bank of Scotland

Switzerlands UBS

US-based GE Capital

Credit Suisse Group

Merrill Lynch is having a joint venture


in Indian investment ban king space DSP
Merrill Lynch. Goldman Sachs holds stakes in
Kotak Mahindra arms. GE Capital is also having
a wide presence in consumer finance through
GE Capital India.
Indias GDP is seen growing at a robust
pace of around 7% over the next few years,
throwing up opportunities for the banking sector
to profit from.
The credit of banks has risen by over
25% in 2004-05 and the growth momentum is
expected to continue over the next four to five
years.
Participation in the growth curve of the
Indian economy in the next four years will
provide foreign banks a launch pad for greater
business expansion when they get more freedom
after April 2009.

THE FIRST, THE OLDEST, THE LARGEST AND THE BIGGEST BANK IN INDIA
The first bank in India to be given an ISO Certification

Canara Bank

The first bank in Northern India to get ISO 9002 certification for their
selected branches

Punjab and Sind Bank

The first Indian bank to have been started solely with Indian capital

Punjab National Bank

The first among the private sector banks in Kerala to become a scheduled
bank in 1946 under the RBI Act.

South Indian Bank

Indias oldest, largest and most successful commercial bank, offering the
widest possible range of domestic, international and NRI products and
services, through its vast network in India and overseas

State Bank of India

Indias second largest private sector bank and is now the largest scheduled
commercial bank in India

The Federal Bank Limited

Bank which started as private shareholders banks, mostly


Europeans shareholders

Imperial Bank of India

The first Indian bank to open a branch outside India in London in 1946
and the first to open a branch in continental Europe at Paris in 1974

Bank of India, founded in


1906 in Mumbai

The oldest Public Sector bank in India having branches all over India and
serving the customers for the last 132 years

Allahabad Bank

The first Indian commercial bank which was wholly owned and managed
by Indians

Central Bank of India

TAG LINES OF SOME BANKS

Banks

Tag Line

Allahabad Bank

A tradition of trust

Andhra Bank

Much more to do with you in Focus

Bank of Baroda

Indias International Bank

Bank of India

Relationships beyond Banking

Bank of Maharashtra

One family one bank

Canara Bank

Its easy to change for those who you, Love together we can do

Central Bank

Build a better like around us

Dena Bank

Trusted Family Bank

10
Indian Bank

Taking banking technology to the common man

Indian Overseas Bank

Good people to grow with

Oriental bank of Commerce

Where every individual is committed

Punjab National Bank

The name you can bank on

Syndicate Bank

Your faithful & friendly financial partner

State Bank of India

With you all the way

Union Bank of India

Good people to bank with

UCO Bank

Honours your trust

United Bank of India

The Bank that begins with U

Vijaya Bank

A friend you can bank on

OLDER PRIVATE SECTOR BANKS


Bank of Rajasthan

Dare to dream

Federal Bank

Your perfect banking partner

J & K Bank

Serving to empower

Karnataka Bank

Your family bank across India

Karur Vysys Bank

Smart way to bank

Laxmi Vilas Bank

The changing face of prospserity

Tamilnad Mercantile bank

Customer oriented and committed to excellence

Nainital Bank

Banking with personal touch

NEW PRIVATE SECTOR BANKS


HDFC Bank

We understand your world

ICICI Bank

Hum Hai Na

Yes Bank

Experience our expertise

INTERNATIONAL BANKS
HSBC

The worlds local bank

Citibank

Citi never sleeps (not used on the Indian website though)

BNP Parisbas

The bank of a changing world

11

Lloyds TBS

for the journey


Founder : Maharaja Bhupinder Singh

STATE BANK OF INDIA

Nationalization : 1959

Old Name: Bank of Calcutta (1809), Bank of


Bengal (1809) and Imperial Bank of India (1921)

Head Office : Patiala (Punjab)

Estd. In Year: 1806


Founder: Bengal Government
Nationalization : 1955
Head Office: Mumbai (Maharashtra)
Share Capital: 59.73 per cent (RBI)
Capital: Rs. 4812 Crores
SBI ASSOCIATES
STATE BANK OF BIKANER AND JAIPUR
Old Name : The Govind Bank Private limited
Nationalization : 1959
Head Office : Jaipur (Rajasthan)
Share Capital : 75 per cent (SBI)
STATE BANK OF HYDERABAD
Old Name : Hyderabad
Estd. In year :1941
Nationalization : 1959
Head Office : Hyderabad (Andhra Pradesh)
Share Capital : 100 per cent (SBI)
STATE BANK OF PATIALA

Share Capital : 100 per cent (SBI)


STATE BANK OF MYSORE
Old Name : Bank of Mysore Ltd.
Estd . in Year : 1913
Founder : M. Visheshwaraiya
Nationalization : 1959
Head Office : Bangalore (Karnataka)
Share Capital : 92.33 per cent (SBI)
STATE BANK OF TRANVANCORE
Old Name : Travancore Bank Ltd.
Estd. in Year : 1945
Nationalization : 1959
Head Office Trivendrum (Kerala)
Share Capital : 75 per cent (SBI)
OTHER NATIONALISED BANKS
ALLAHABAD BANK
Estd. in Year : 1865
Nationalization : July 19, 1969
Head Office : Kolkata (W. Bengal)

Old Name : Patiala State Bank

Share Capital : 55. 23 per cent (Central


Government)

Estd. in Year : 1917

ANDHARA BANK

12
Estd . in Year : 1923

Share Capital
Government )

Founder : P. Sitaramaiya

Head Office : Hyderabad (Andhra Pradesh)


:

62.50

per

cent

(Central

per

cent

(Central

CANARA BANK
Old Name : Canara Bank Hindu Permanent Fund
Estd. in Year : 1906

Capital : Rs. 48 Crores

Founder : A. Subbarao Pai

BANK OF BARODA

Nationalization : July 19, 1969

Estd. in Year : 1908

Head Office : Bangalore (Karnataka)

Founder : Maharaja Sayajirao-III

Share Capital
Government)

Nationalization : July 19, 1969

66.81

: 73.17 per

cent

per

cent

(Central

CENTRAL BANK OF INDIA


Estd in Year : 1911

Capital : Rs. 619 Crores

Founder : Sorabjee Pochkhanwala

BANK OF INDIA

Nationalization : July 19, 1969

Estd. in Year : 1906

Head Office : Mumbai (Maharashtra)

Founder : businessmen of Bombay

Share Capital : Rs. 814 Crores

Nationalization : July 19, 1969

CORPORATION BANK

Head Office: Mumbai (Maharashtra)


Share Capital:
Government)

(Central

Capital : Rs. 1125 Crores

Head Office : Baroda (Gujarat)


Share Capital
Government)

76.77

Capital : Rs. 280 Crores

Nationalization : April 15, 1980

Share Capital
Government)

69.47

per

Capital : Rs. 1554 Crores


BANK OF MAHARASHTRA
Estd. in year : 1935
Nationalization : July 19, 1969
Head Office : Pune (Maharashtra)

cent

Estd. in Year : Corporoation Bank


(Central

Nationalization : April 15, 1980


Head Office : Mangalore (Karnataka)
Share Capital
Government)

57.17

per

cent

(Central

Capital : Rs. 206.8 Crores


DENA BANK
Old Name : Devkaran Nanji Banking company
Ltd.

13
Estd. in Year : 1938

PUNJAB AND SINDH BANK

Founder : Pran Lal Devkaran Nanji

Estd. in Year : 1908

Nationalization : July 19, 1969

Nationalization : April 15, 1980

Head Office : Mumbai (Maharashtra)

Head Office : New Delhi

Share Capital
Government)

51.10

per

cent

(Central

PUNJAB NATIONAL BANK


Estd. in Year: 1895 (Lahore)

Capital : Rs. 591 Crores

Nationalization : July 19, 1969

INDIAN BANK

Head Office : New Delhi

Estd. in Year : 1907

Share Capital
Government)

Nationalization: July 19, 1969


Head Office : Chennai (Tamil Nadu)

57.79

per

cent

Capital : Rs. 119 Crores

Capital : Rs. 418 Crores

SYNDICATE BANK

INDIAN OVERSEAS BANK

Estd. in Year : 1925

Estd. in Year : 1937

Founder : U.S. Pai, Vaman Kudwa, and T.M.A.


Pai

Founder : M. Chidambaram Chettiyar

Nationalization : July 19 1969

Nationalization : July 19, 1969

Head Office : Manipal (Karnatala)

Head Office : Chennai (Tamil Nadu)


Share Capital : 75 per cent (Central Government)
Capital : Rs. 319 Crores

Share Capital
Government)

73.52

per

cent

UCO BANK

Estd. in Year : 1943

Old Name : United Cormercial Bank Ltd.

Founder : Ram Bahadur Lala Sohan Lal

Estd. in Year : 1943

Nationalization : April 15, 1980

Nationalization : July 19,1969

Head Office : New Delhi

Head Office : Kolkata (W. Bengal)

:66.5

Capital : Rs. 327 Crores

(Central

Capital : Rs. 425 Crores

ORIENTAL BANK OF COMMERCE

Share
Capital
Government)

(Central

percent

(Central

Share Capital:
Gorvernment)

74.98

Capital : Rs. 810 Crores

percent

(Central

14
UNION BANK OF INDIA

Functions of a Bank

Estd. in Year : 1919

a. Receipt of Deposits: Bank collects 3 types of


deposits-current Deposits. Saving Deposits and
Fixed Deposits.

Nationalization : July 19,1969


Head Office : Mumbai (Maharashtra)
Share Capital
Government)

60.85

per

cent

(Central

c. Agency Services

Capital : Rs. 1375 Crores

I. Collection of bills, promissory notes and


cheques

UNITED BANK OF INDIA


Old Name : United Bank of India

II. Collection of interest, dividends, premium etc.

Estd.in Year : 1950

III. Purchase and sale of shares and securities

Nationalization : July 19, 1969

IV. Acting as a trustee

Head Office : Kolkata (W. Bengal)

d. General Services

Capital : Rs. 277 Crores

I. Issue of letters issue of credit, travelers


cheques, Bank drafts, circular notes.

VIJAYA BANK

II. Safety Deposits

Estd.in Year : 1931

III. Supplying trade information

Founder : A.B. Shetty

Commercial Banking in India.

Nationalization : April 15, 1980


Head Office : Bangalore (Karnataka)
Share Capital
Government)

b. Lending of Money: These are different form of


lending cash credits, overdrafts, loans and
advances or discounting of bills of exchange

53.87

per

cent

(Central

Capital : Rs. 77.5 Crores

COMMERCIAL BANKS
Meaning of Banks: Banks are the institutions that
mobilise the savings of the community and
make
them available to the entrepreneurs.
They acts as a bridge between the users of
capital and those who save but cant use the
funds themselves.

A. Government nationalized 14 major commercial


Banks with effect from 1969
B. Six more banks were nationalized in 1980
C. Two Banks were merged in 1993, so at present,
there are 19 nationalised banks
Nationalisation was taken place due to following
reasons.
A. Private ownership of commercial banks and
concentration of economic power
B. Urban-bias
C. Neglect of agricultural sector
D. Violation of Norms

15
E. Speculative activities
F. Neglect of priority sectors.
Progress of Commercial Banks.
A. Expansion of Branches: Mostly all the banks
has opened their branches in whole country.
B. Branches in Rural Area: Number of rural
branches was rose to 42% in 2005 as compare
to 22% in 1969.
C. Deposit mobilization: Maharashtra leads all
other states and accounts for more than 1/5 th of
the aggregate deposits.
D. Bank Lending: In 1969 agriculture, small
scale industries and small retail trade accounted
for about 14% of the commercial bank credit
but now in May 2004 it rises to 40%.
Shortcomings of Commercial Banks
A. In rural areas 75% of the population resides
but they had only 50% of the bank branches.
B. There are regional imbalances.
C. Almost 50% of the loans and advances of
the banks have not been recovered.
D. Quality of services had been deteriorated.
A. Commercial Banks refer to both scheduled
and non-scheduled commercial banks which are
regulated under Banking Registration Act. 1949.
a) Scheduled Commercial Banks are grouped
under following categories;
1. State Bank of India nad its Associates
2. Nationalised Banks
3. Foreign Banks
4. Regional Rural Banks
5. Other Scheduled Commercial Banks

b) Non-Scheduled Commercial Banks


Note: Banks in the groups (1) & (2) above are
known as public sector banks whereas, other
scheduled commercial banks mentioned at group (5)
above are known as private sector banks.
B. Bank Offices comprise of branches doing
banking business (i.e. accepting deposit
and / or offering credit their customers) and
administrative offices.
C. Branches of banks refer to those offices
which are engaged in either;
1. Banking business (i.e., either accepting
deposit and/or offering credit to their
customer); or
2. Banking and foreign exchange business; or
3. Administration,
exchange; or

banking

&

foreign

4. Administration and banking; or


5. Administration and foreign exchange; or
6. Only foreign exchange business.
D. Administrative Offices include head office,
zonal office, regional office, local head
office, training centre, clearing cell, service
branch, asset recovery branch, divisional
office etc,.
E. Banked Centre is a centre, which has at
least branch or office of commercial or cooperative bank.
F. Unbanked Centre is a centre in which no
branch of any commercial/co-operative bank
is functioning.
G. Population group of banked centres are
defined as below:
1. Rural group includes all banked centres
with population of less than 10,000.

16
2. Semi-urban group includes banked
centres with population of 10,000 and above
but less than 1 lakh.
3. Urban group includes banked centres with
population of 1 lakh and above but less than
10 lakh.
4. Metropolitan group includes banked
centres with population of 10 lakh and
above.

other Link Office


C

Handling Forex through A Or B Category


Office

J. Distrct-wise list
District-wise list of branches/ offices are based
on latest available information on districts, inclusive
of details available on formation/ re-organisation of

H. Type
of
Business
activity
wise
Designatory Code of Branches / Offices of
banks
Code

Description

districts.

MONETARY POLICY
Monetary policy means the policy of the central

Only Banking Business,

bank which it introduced with the object to

Banking & Foreign Exchange Business,

administer and control the countrys money supply

Administration, Banking & Foreign


Exchange Business,

including currency, demand deposits and foreign

Administration & Banking Business

According to A.G. Hart, Monetary Policy is a

Administration & Foreign Exchange


Business

policy which influences the publics stock of

Only Foreign Exchange Business,

assets, or both that is policy which influences the

Only Administration/ Training /etc.

public liquidity position

Non-Scheduled Banks doing Banking


Business

OBJECTIVE OF MONETARY POLICY

exchange rates.

money substitutes or the public demands for such

Different economists have different views on the


I. Authorised Dealer (AD) Branches:
Branches of banks authorized to deal in foreign
exchange business. Authorised Dealer Category
(AD-Category):
Code

objectives of monetary policy because they keep on


changing from time to time as per the change in
business activities and the level of economic
development. The main objectives of the monetary
policies are as follows:

Description

Maintaining Independent Foreign Currency


(FC) Accounts

Maintaining FC A/Cs. Through H.O. Or any

1) Stability of exchange rates 2) Full employment


3) Price stability 4) Neutrality of money 5) High rate

17
of economic growth 6) Encourages saving and

an instrument of monetary policy. The purpose of

capital formation

open market operations are to contract or expand the

LIMITATION OF MONETARY POLICY

supply of reserves of banks, thus contract their


power to expand credit and to provide an orderly

Though, the monetary policy is very important

market for government securities or to influence the

for the development of the country, but it has got

interest rates through the quantitative effect on

certain limitation too. These limitations are as

reserves.

follows.
iii. Change in reserve requirements: The two
1) Underdeveloped capital and money Market
2) No integrated rate of interest structure

types of reserves that banks need to maintain.


i.

CRR: Cash Reserve Ratio refers to that

3) llliteracy and social obstacles.

portion of total deposits which a commercial

4) Lack of cooperation among the banks.

bank has to keep with the Central Bank in


the form of cash reserves.

5) Banking habits
6) Existence of black money.
7) Government policies.
Monetary policy is implemented by the RBI
through the instruments of credit control. Generally

ii.

SLR: Statutory Liquidity Ratio refers to that


portion of total deposits which a commercial
bank has to keep with itself in the form of
cash reserves. This will tend to contract
credit in the system and vice versa.

2 types of instruments are used to control credit.


2. Qualitative Measures :
1. Quantitative Measures :
i. Bank Rate: Bank Rate is the minimum rate
charged by the bank for discounting approved bills
of exchange. It is a rate at which the Central Bank is
prepared to rediscount the bill or lend money to the
commercial bank or other financial institutions. By
effecting a change in the bank rate, the Central Bank
can effectively control the credit system.
ii. Open market operation: The open market
operation has been defined as the purchase and sale
of government and other eligible securities by the
Central Bank in order to check the flow of credit. As

i. Moral suasion: This method involves


advice regulation and persuasion by the Central
Bank to the commercial banks to cooperate with the
Central Bank in implementing the credit policy. The
Central Bank in this method merely uses its moral
influence on commercial bank and acts as a
supplement to other methods of credit control.
ii. Fixation of margin requirement: The
margin is the difference between the loan value and
the market value. The Central Bank fixes the margin
rate. If it is 40% that means that the commercial
bank can lend only up to 60% of the market value of

18
the securities. Thus, by changing the margin rate the
Central Bank controls credit.
iii. Credit rationing: It means that the central
bank fixes a limit upon its rediscounting facilities for
any bank. It also means that Central Bank fixes the
quota of every affiliated bank for financial
accommodation from the central bank.
iv. Regulation of consumer credit: This
method proves useful when economy faces the
problem of inflation. Under this method the Central
Bank curtails or extends the limit to which
commercial banks can provide credit/ finance
facility to the consumers or durable goods.
v. Publicity: Under this method the Central

3) Only those who are taking housing loans


4) All (1), (2) and (3) above
5) None of these
2. Which of the following agencies/ organizations
in India maintains the Micro Finance
Development and Equity Fund which was in
news recently?
1) Confederation of Industries in India
(CII)
2) Indian Banks Association (IBA)
3) Small Industries Development Bank of
India (SIDBI)
4) Reserve Bank of India (RBI)
5) National Bank for Agriculture and Rural
Development (NABARD)

Bank gives wide publicity to what is good and what


is bad in the credit system of the country.
vi. Direct actions: This method involves
issuing general instruction by the Central Banks to
all the commercial banks and implies the use of

3. As per news in various newspapers kfw


Group released another installment of its
financial aid to India. kfw is an organization /
bank based in __
1) France

coercive action against those commercial banks

2) Japan

whose credit policies do not confirm to the policies

3) Italy

directed by Central Bank.

OBJECTIVE QUESTIONS ON
BANKING INDUSTRY
1. The Business Correspondence Model being
adopted by the banks provides Banking
facilities to which of the following

4) China
5) Germany
4. Which
of
the
following
agencies/
organizations has decided to make major
changes for ULIPs?
1) IRDA

1) Only Corporate borrowers

2) RBI

2) Only weaker sections the society and


people of small villages

3) AMFI
4) FRBI

19
5) None of these
5. Which of the following is NOT a financial
term?
1) Ac:d Test
2) Double Fault
3) Gross Profit
4) Depreciation
5) Cash flow
6. In the terms of economics, the recession
occurring two times with a small gap in
between is known as_
1) Double Deflation
2) Deflation
3) Deep Recession
4) Double Dip Recession
5) None of these
7. A consortium of commercial banks has joined
hands to float a new company which will
collect data related to frauds and security risks.
Which of the following is the name of the
company?

Japan had to take this special measure which


surprised many countries?
A) Japan was fearful that a rising currency would
damage its export/led recovery plan.
B) It was one of the measures taken by Japan to
come out of its economic setback it suffered in
the recent past.
C) Since the US Dollar has became a Weak
Global Currency, Japan wants the Yen to be
recognized as the global currency in place of
the US dollar.
1) Only (A)
2) Only (B)
3) Only (C)
4) Only (A) and (B)
5) Only (B) and (C)
9. The Reserve Bank of India has set up a
Working Group to review current operating
procedure of monetary policy including LAF.
What is full form of LAF?
1) Liquid Assets and Funds
2) Liabilities and Funds

1) Loss Data Consortium

3) Liabilities Adjustment Finance

2) Customer Service Corporation

4) Liquidity and Finance

3) BANK NET India Ltd.

5) Liquidity Adjustment Facility

4) Nu tech corporate Services Ltd.


5) None of these
8. As per newspaper reports the Bank of Japan,
which is the Central Bank of the Country, took
special measures, first time in last six years, to
contain the rise in the value of Yen. Why did

10. Which of the following organizations/ agencies


plays a major and crucial role to moderate &
control inflation in a country?
A) Central Bank of the country
B) Stock Markets of the country
C) Planning Commission the country

20
1) Only (B)
2) Only (A)
3) Both (A) and (B)

5) New Deposits and Term Liquidity


13. The Reserve Bank of India recently decided to
review its monetary policy more frequently.
This measure of the RBI will be called RBIs

4) Both (B) and (C)

A) Monetary Measure

5) All (A), (B) and (C)

B) Financial market Measure

11. Many times we read about balance of


Payment in economic literature. Which of the
following statement (s) define the term
correctly?
A) It is a record of all transactions made
between one particular country and all other
countries.
B) It is a record of all transactions made
between the countries wherein the payment
was made in a currency other than US
Dollar.
C) It is a record of all exports/ imports from/ by
a country and difference in amount of both
in terms of payment in US dollars.
1) Only (A)
2) Only (C)
3) Only (B)
4) Both (A) and (B)
5) Only (B) and (C)
12. As per the existing policy the Cash Reserve
Ratio (CRR) of scheduled banks is fixed at a
certain percentage of their NDTL. What is full
form of NDTL?

C) Regulatory Measure
1) Only (A)
2) Only (B)
3) All (A), (B) and (C)
4) Both (A), and (C)
5) Only (C)
14. Basel Committee on Banking Supervision is
also/or popularly known as__
1) Thomas Cook Committee
2) Talwar Committee
3) Bank
for
committee

3) National Deposits & Total Liquidity


4) Net Duration & Total Liquidity

Settlements

4) Peter Cook Committee


5) Banking Regulation Committee
15. Which of the following is/ are the highlights of
the Foreign Trade Policy 2009-14. Which was
reviewed by the government recently?
A)

Government to increase exports up to


US $ 200 billion this fiscal year.

B)

List of items allowed for duty free


imports reduced by 30%.

C)

Few towns like Barmer, Bhiwandi and


Agra were declared towns of Export
Excellence.

1) New Demand & Tenure Liabilities


2) Net Demand & Time Liabilities

International

1) Only (A)

21
2) Only (B)

1) Medium Alternate Tax

3) Only (C)

2) Maximum Alternate Tax

4) Only (A) and (C)

3) Minimum Alternate Tax

5) Only (A) and (C)

4) Mild Alternate Tax

16. The Direct tax Code which was revised


recently will be implemented from which of the
following dates?
1) April 2011

20. Many times we read about SWIFT in


newspaper. What is its full form?
1) Sound
Worldwide
transactions

2) April 2012
3) January 2011

2) Safe Window
transaction

4) January 2012

In

Institution
case

of

for

financial

3) Society for Worldwide Interbank Financial


Telecommunication

5) None of these
17. Government may reduce its budgetary deficit
by various measures. Which of the following is
NOT one such measure?

4) Safe Window for Interbank financial


Transactions
5) None of these

1) Market borrowing

21. In the world of commodity exchanges- the


concept of A standard contract to buy or sell a
specified commodity at a certain date in future
and at a market driven price- is known as____

2) Deficit financing
3) Foreign grants
4) Increasing taxes

1) Future contract

5) Reducing taxes and increasing subsidies


18. Commercial Papers are issued by__

2) Forward contract
3) Market contract

1) a company to a bank

4) Standard contract

2) Banks to Companies

5) None of these

3) Banks to Banks

22. The foreign trade policy in India is announced


by the____

4) Company to its suppliers

1) Reserve Bank of India

5) None of these
19. MAT provision
for_____

5) None of these

in

income

Tax

stands

2) Ministry of Commerce
3) FEDAI

22
4) Ministry of Finance
5) None of these
23. Lending to Self Help Groups (SHGs) by Banks
is known as_
1) Non priority Sector
2) Priority sector

3) Balance between price of all essential


commodities
4)

Balance between growth price stability and


financial stability

5)

None of these

26. Which of the following is/ area / an indicator of


the financial health of a country?

3) Commercial sector

a) GDP

4) Corporate Sector

b) PPP

5) None of these

c) FDI in a year

24. Through which one of the following source(s),


Indian companies are allowed to access funds
from abroad?

1) Only (A)

A) External Commercial Borrowings (ECB)


only

3) Only (C)

B) Foreign Currency Convertible Bonds


(FCCBs)/.
Foreign
Currency
Exchangeable Bonds only
C) Preference Shares Only

2) Only (B)

4) All (A), (B) and (C)


5) None of these
27. Stock Exchanges play a role in an economy
which may be termed as__

1) only (A)

1) hardly useful

2) Only (B)

2) useful but need strict regulation

3) All (A), (B) and (C)

3) harmful to proper capital markets

4) Only (A) and (C)

4) a gambling-where many investors get


ruined

5) Only (C)
25. While tackling the problem of inflation. Which
one of the following aspects is taken into
consideration by the RBI?
1)

Balance between budget deficit and


price stability

2)

Balance between growth and financial


stability

5) a very
inflation

important

segment

regulate

28. Which of the following is/ are called Key


policy rates by Reserve Bank of India?
A) Bank rate
B) CRR
C) SLR

23
1) Only (A)
2) only (B)
3) Only (B) and (C)
4) Only (A) and (B)
5) All (A), (B) and (C)
29. As per the reports published in various
newspapers, the Reserve bank of India has
directed all core investment companies in India
to register themselves with it if their asset size
is ` 100 crore or more. Which of the following
is true about the Core investment companies
(CICs)?
A) CICs are non-banking finance companies.
B) CICs invest in shares for the purpose of
taking stake in a company rather than for
trading.
C) CICs are required to register with RBI as
most of them were found involved in
business in foreign countries and not in
India.
1) Only (A)

5) None of these
31. Which of the following terms is NOT used in
the field of banking and finance?
1) Blue chip companies
2) Commercial paper
3) Hedge
4) Premium Bonds
5) Wobbe Index
32. Which one of the following advice has been
given by the Finance Minister of India to
Banks, while addressing the 5 th International
Banking and Finance Conference?
1)

Banks should provide ATMs in unbanked


areas

2)

Banks should extend loans to the people


in rural areas

3)

Banks should relax security norms for


small businesses

4)

Banks should prepare a road map for


financial inclusion covering rural
households

5)

None of these

2) Only (A) and (B)


3) Only (B) and (C)
4) All (A), (B) and (C)
5) None of these
30. As per newspaper reports, the RBI recently
divested it stake in NABARD. Now RBIs
stake in it is only________

33. Expand the term IFRS__


1) Indian Financial Review Standards
2) Indian financial Reporting Standards
3) International financial Review Standards

1) 10%

4) International
Standards

2) 5%

5) None of these

3) 3%
4) 1%

Financial

34. What is a corporate Hedging?

Reporting

24
1) It is a mechanism to protect firms exposure
to equity risk

1) is an easy way of raising capital from a


factoring company by small business.

2) It is a mechanism to protect firms exposure


to market risk

2) is selling of account receivables on a


contract basis for cash payment to a factor
before it is due.

3) It is a mechanism to protect firms exposure


to forex risk
4) It is a mechanism to protect firms exposure
in international trade
5) None of these
35. India is following a system of flat money?
1) The currency is backed by Government
Guarantee

3) is an arrangement for raising short term


money against prepaid expenses.
4) is a method of discounting of long term
bills.
5) None of these
38. Under provisions of which one of the following
Acts, the RBI issues directives to the Banks in
India?

2) The currency is backed by tangible assets

1) RBI Act

3) The currency is backed by gold reserves

2) Banking Regulation Act

4) The currency is backed by budgetary


support

3) Essential Commodities Act

5) None of these
36. A lease
1) is a rental agreement between owner of a
flat/house and the Tenant, whereby the
Tenant pays specified amount on specified
dates to the owner.
2) is a legal contract-between the owner of an
asset and its user, containing many
conditions including payment of a specified
amount by the user to the Owner for use of
the asset.
3) is mainly used for machinery.

4) RBI and Banking regulation Act


5) None of these
39. Which one of the following tools is used by
RBI for selective credit control?
1) It advises banks to lend against certain
commodities
2) It advises banks to recall the loans for
advances against certain commodities
3) It advises banks to charge higher rate of
interest for advance against certain
commodities

4) is one of the ways resorted to by the


companies.

4) It discourages certain kinds of lending by


assigning higher risk weights to loans it
deems undesirable

5) None of these

5) None of these

37. Factoring____

40. For which one of the following Loan Products


teaser loans are offered by Banks?

25
1) Education Loans
2) commercial Loans

44. Which one of the following may be the


consequence of buying forex in the market by
the RBI?

3) Loans against security of gold

1) It leads to inflation

4) retail trade Loans

2) It leads to control over inflation

5) Home Loans

3) It does not affect inflation

41. Which one of the following has decided to give


guarantee for infra loans?
1) IDBI
2) SBI
3) IIFCL
4) SIDBI
5) None of these
42. Which one of the following has got RBI nod to
issue pre-paid cards to its clients?

4) It results into deflation


5) None of these
45. Under provisions of which one of the following
Acts, the Reserve Bank of India has the power
to regulate, supervise and control the banking
sector?
1) RBI Act
2) Banking regulation Act
3) Negotiable Instrument Act

1) LIC

4) RBI and Banking Regulation Act

2) GIC

5) None of these

3) SIDBI
4) NABARD
5) None of these
43. As per recent newspaper reports, Indias public
debt rises 2.8 percent to ` 28 lakh crores.
Which one of the following is the reason that
has forced commercial banks to buy
government securities?
1) Lack of credit growth

46. Which of the following has proposed $ 11


billion infrastructure debt fund?
1) NABARD
2) SBI
3) RBI
4) Ministry of Finance, Govt. of India
5) Planning commission
47. The NAV of a mutual fund________

2) Increase in credit growth

1) Is always constant

3) Raising of repo rate

2) Keeps going up at a steady rate

4) Raising of reverse repo rate

3) Fluctuates with market price movements

5) None of these

4) Cannot go down at all

26
5) None of these
48. A saving Bank Deposit Account is one
where_____
1) Amounts are deposited and are withdrawn as
per requirement of the customers.
2) The deposits are made only once in a year

3) Ministry of Finance
4) SEBI
5) IRDA
52. The regulator of the banking system in India is
__
1) AMFI

3) Periodical fixed amount are deposited month


wise and withdrawals are allowed after a
fixed period

2) Finance Minister

4) All of the above

4) IBA

5) None of these

5) RBI

49. Which of the following is a financial asset?

3) SEBI

53. Ten rupee notes contain the signature of_____

1) Gold

1) Finance Secretary, GOI

2) Silver

2) Chairman, State bank of India

3) Shares in a demat account

3) Governor, Reserve Bank of India

4) Land & buildings

4) Finance Minister, GOI

5) None of these

5) Prime Minister

50. Zero balance account is generally allowed to be


opened for____

54. Loan against the security of immovable


property is by executing an agreement of ___

1) Salaried class

1) Assignment

2) businessmen

2) Pledge

3) foreign customers

3) Transfer

4) children

4) Mortgage

5) Senior citizens

5) None of these

51. In deposits Account KYC (Know Your


Customer) has been implemented in 2002 as
per directive of _____
1) IBA
2) RBI

55. Credit Rating______


1) is used to rate the borrowers while giving
advances.
2) is used to work out performance of the
employees.

27
3) is used to calculate the number of excellent
audit rated branches.

how many days of the closure of the Initial


Public Offers (IPOs))?

4) is NOT used in any BANK.

1) 60 days

5) is necessary before giving promotion to


employees

2) 12 days

56. In Corporate Hedging, which one of the


following types of risks can be covered?

3) 30 days
4) 45 days

1) Liquidity risk

5) None of these

2) Currency risk

60. RTGS stands for___

3) Credit risk

1) Real time gross Settlements

4) Transaction risk

2) Reduced Time Gross Settlements

5) None of these

3) Relative Time Gross Settlements

57. Bank of Rajasthan is now merged with which


of the following Banks?
1) AXIS Bank

4) Real total Gross Securities


5) None of these
61. DSCR indicates the ability of a company to ___

2) IDBI Bank

1) meet its current liabilities

3) ICICI Bank

2) service its shareholders

4) HDFC Bank

3) meet its long term debt obligations

5) None of these

4) raise further capital

58. Which of the following is the full form of the


abbreviation
IFRS
as
used
in
corporate/finance sector?
1) Indian Financial Reconciliation Standards
2) Interpretation and Formal reporting System
3) International Financial Reporting Standards
4) International Financial Reporting System
5) None of these
59. As per the new guidelines issued by SEBI,
companies are required to list shares within

5) None of these
62. An IPO is ____
1) initial price offered by a private limited
company to its shareholders
2) an offer by an unlisted company for sale of
its shares for the first time to the public
3) used to increase the share capital of an
unlisted company
4) a book building process
5) None of these

28
63. A decreasing current ratio indicates__
1) a stable liquidity
2) an increasing liquidity

66. The RBI reviews its credit and monetary policy


at regular intervals and also in between. What
is the purpose of the same?

3) a strained liquidity

A) To ensure that inflation does not cross the


limit.

4) satisfactory current solvency

B) to ensure that banks have enough liquidity

5) None of these

C) To ensure that cost of the fund does not


reach a very high level

64. Private equity investors invest in a company


based mainly on _______

1) Only (A)

1) the age of the company

2) Only (B)

2) the location of the company

3) All (A), (B) and (C)

3) the activity undertaken by the company

4) Only (B) and (C)

4) the creditability and the valuation of the


company

5) None of these
67. The Reserve Bank does not have an office in__

5) the existing profitability of the company

1) Kanpur

65. Financial ratio analysis of a company helps is


understanding___

2) Jaipur

A) the long term debt raising capacity


B) the short term solvency
C) the reasons for fluctuations in its stock
prices
D) the net worth of the company
E) the profitability
1) A, B, C, D
2) B, C, D
3) C, D, E
4) C, A, B, D
5) A, B, D, E

3) Lucknow
4) Bhopal
5) Gulbarga
68. Which one of the following is the objective of
Corporate Social Responsibility (CSR) issued
by the Government?
1) To ensure that Companies take steps for
rehabilitation of project affected people
2) To ensure that Companies use some of their
profits for educating the project affected
people
3) To ensure that Companies provide health
care to project affected people
4) To ensure that Companies take preventive
measures to avoid environmental damage

29
5) None of these
69. Which one of the following have provided
bailout package to Ireland to deal with its debt
crisis?
1) EU only
2) IMF only
3) World Bank Only
4) Only (1) and (2) above
5) All (1), (2) and (3) above
70. Which one of the following has launched
guidebook on standards for exporters?
1) RBI
2) SEBI
3) IBA
4) UNIDO
5) UNESCO
71. Which of the following banks has taken over
the Centurion Bank of Punjab?
1) ICICI bank
2) IDBI Bank
3) HDFC Bank
4) AXIS Bank
5) None of these
72. Which of the following nation is considered the
originator of the concept of Micro Finance?
1) India
2) Bangladesh
3) South Africa

4) USA
5) None of these
73. What is the full form of ULIP, the term which
was in the news recently?
1) Universal Life & Investment Plan
2) Unit Loan & Insurance Plan
3) Universal Loan & Investment Plan
4) Uniformly Loaded Investment Plan
5) Unit Linked Insurance Plan
74. The Main function of I. M. F. is to
1) Finance investment loans to developing
countries
2) Act as a private sector lending arm of the
world Bank
3) Help to solve balance of payment problems
of member countries
4) Arrange international deposits
5) None of these
75. RBI has asked banks to make a plan to provide
banking services to all villages having a
population upto 2000. This directive issued by
the RBI will fall in which of the following
categories?
1) Plan for Financial Inclusion
2) Efforts to meet the targets of Priority Sector
Lending
3) Extension of Relief Packages to the Farmers
4) Plan for opening more rural branches
5) None of these
76. Which of the following is not a part of the
scheduled banking structure in India?

30
1) Money Lenders
2) Public sector banks

80. By which one of the following years,


Government proposes to introduce IFRS
norms?

3) Private Sector Banks

1) April 2011

4) Regional rural Banks

2) October 2011

5) State Co-operative banks

3) April 2012

77. The rate of interest on Savings Bank Account is


stipulated by
1) The concerned bank
2) RBI
3) Indian Banks Association

4) October 2012
5) None of these
81. At which one of the following rate, the central
bank lends to banks against government
securities?

4) Government of India

1) Repo Rate

5) Banking Codes and Standards Board of


India

2) Reverse Repo rate

78. Many times we read a term CBS used in


banking operation. What is the full form of the
letter C in the term CBS?
1) Core
2) Credit
3) Continuous
4) Complete
5) None of these
79. Which one of the following is a driving force
influencing the industrial growth of an
economy?
1) Economic Factors only
2) Investment only
3) Innovation/Market Base only
4) Only (1) and (2)
5) All (1), (2) and (3)

3) Bank Rate
4) SLR
5) CRR
82. As per the reports published in the newspapers
the banks, particularly public sector banks are
tying up with various rating agencies for
providing a qualitative assessment of the credit
needs of the borrowers. Which amongst the
following is/ are such credit rating agencies in
India?
A) CARE
B) CRISIL
C) ULIP
1) Only A
2) Only B
3) A and B only
4) Only C
5) All A, B and C

31
83. As per the newspaper reports the RBI is
planning to introduce plastic Currency
Notes. What is/ are the benefits of Plastic
Notes ?
A) Their Shelf life will be longer.
B) It will replace plastic money or credit, debit
cards which are giving birth to many
fraudulent practices.
C) Printing will be cheaper.
1) Only C
2) Only B
3) Only A
4) All A, B and C
5) None of these
84. Sub Prime Lending is a term applied to the
loans made to_
1) those borrowers who do not have a good
credit history.
2) those who wish to take loan against the
mortgage of tangible assets.
3) those who have a good credit history and are
known to bank since 10 years.
4) those borrowers who are most preferred
customers of the Bank.
5) None of these
85. As per the reports published in various journals
and newspapers the small borrowers in rural
areas still prefer to take informal route for their
credit needs. Which of the following is the
informal route of credit in financial sector?
1) Credit cards
2) Loan against gold from financial institute

3) Debit cards
4) Money lender
5) None of these
86. Basel-II norms are associated with which of the
following aspects of the banking industry?
1) Risk management
2) Manpower planning
3) Retirement benefits for the employees
4) Corporate Governance
5) None of these
87. What is meant by Underwriting the term
frequently used in financial sector?
1) Under valuation of the assets
2) The Act of taking on a risk for a fee
3) Giving a Guarantee that a loan will not
become a bad loan
4) The act of permission to float an IPO
5) None of these
88. As we have noticed many banks of Indian origin
are opening offices/branches in foreign
countries. Why is this trend emerging at a very
fast pace?
A) These banks wish to provide banking
facilities to foreigners as banking facilities
are not plenty in many foreign countries.
India wants to take an advantage of the
situation.
B) These banks wish to help Indian firms, to
acquire funds at internationally competitive rates.
C) These banks wish to promote trade and
investment between India and other countries.

32
1) Only A

3) It is buying of one currency and selling of


another currency

2) Only B
3) Only C

4) It is simultaneous buying of one currency and


selling of another currency

4) All A, B and C

5) None of these

5) Only B and C
89. Which of the following
banking/finance related term?

92. Expand the term FLCC


is

NOT

1) Financial Literacy and credit counselling


2) Financial Literacy Communication Centre

1) Credit wrap

3) Financial Literacy Call Centre

2) EMI

4) Fiscal Literacy and Credit counselling

3) Held to Maturity

5) None of these

4) Exposure limit
5) Diffusion

93. What is a fiscal deficit?


1) It is a gap between the values of the Exports
and imports

90. A National development Fund was created by the


government of India a few years ago. What was
the purpose of the fund ?

2) It is a gap between Exports and Imports


minus external borrowings

A) Boost Investment in social sector projects to


promote education, healthcare and employment.

3) It is a gap between total expenditure and total


receipts of the Government

B) Capital investments in selected profitable


PSEs to yield adequate returns.

4) It is a gap between total receipts minus


External borrowing

C) To fund revision in pay structure of central


Government employees.
1) Only A

5) None of these

2) Only B

Which
one
of
the
following
organisations/agencies are involved in drafting
the Union Budget of India?

3) Only C

1) The Planning Commission only

4) Both A and B

2) The Comptroller and Auditor General only

5) None of these

3) Administrative Ministries only

91. What is Forex?


1) it is buying of foreign currency
2) It is selling of foreign currency

94.

4) (1) and (2) only


5) All (1), (2) and (3)

33
95.

Which of the following is NOT a


programme/agency of the Union Ministry of
Finance ?

1) RTGS
2) ASBA

1) Investor Education and Protection Fund


(IEPF)

3) Prefunded Cheques

2) National Foundation
Governance (NFCG)

5) None of these

for

Corporate

3) Serious Frauds Investigation Office (SFIO)


4) Investment Commission
5) Central Rural Sanitation programme
96. The approach of Micro-Credit or Banking with
the poor is comparatively a new concept in the
field of rural credit. This approach has been tried
very successfully in which of the following
countries ?

4) SCSBs

99. The Reserve Bank of India recently advised the


Government of India to spend and loosen the
strings of its purse. How will this benefit the
banking and finance sector ?
A) It will ease liquidity crisis in the market.
B) It will help banks borrow less from the RBI.
C) It will bring down inflation.
1) Only (A) and (B)

1) Bangladesh

2) Only (B)

2) India

3) Only (B) and (C)

3) China

4) All (A), (B) and (C)

4) Japan

5) None of these

5) None of these
97. Which of the following is introduced by banks to
increase financial inclusion?
1) Stimulus package
2) Internet Banking
3) Business correspondent
4) Corporate Banking
5) None of these
98. Which of the following is a type of banking
application which authorizes a bank to black a
specific sum of money in an individuals bank
account to be invested in an Initial Public Offer
(IPO) ?

100. The rate on interest charged by RBI for lending


money to various commercial banks by
rediscounting of the bills in India is called
1) Exchange Rate
2) Statutory Liquidity Ratio
3) Cash reserve Ratio
4) Bank Rate
5) Interest Rate
101. The RBI has asked banks to spell out their
policy, procedures and size of the business on
which of the following aspects of banking ?
1) On-shore banking

34
2) Off-shore banking
3) Investments in secondary market
4) Wealth management
5) None of these
102. Who amongst the following has asked banks to
adopt six step approach to prevent diversion of
funds by the companies ?
1) Company Registrar
2) SEBI
3) RBI

3) II is as per Bank for International Settlement


(BIS) directives
4) It is to protect NBFCs from any impact of
possible economic slowdown
5) None of these
105. For which one of the following reasons Basel II
norms are to be followed by Commercial Banks ?
1) Risk management
2) Adoption
standards

of

international

accounting

4) AMFI

3) Restriction on bonus payments to bank


executives

5) All of the above

4) Transparency In disclosures

103. Through which one of the following sources


domestic funds are raised by companies ?
A) IPO only

5) None of these
106. Which one of the following will set up core
banking infrastructure for rural banks ?

B) FPO only

1) RBI

C) Commercial papers

2) NABARD

1) Only (C)

3) SIDBI

2) All (A), (B) and (C)

4) IBA

3) Only (A) and (C)

5) None of these

4) Only (A)
5) Only (A) and (B)
104. Which one of the following was the reason
owing to which government wants Reserve
Bank of India to tighten prudential norms for
NBFCs ?
1) To reduce liquidity in the market
2) II is as per Basel II requirements

107. Which of the following terms is used in


Banking Field ?
1) Interest Rate Swap
2) Input Devices
3) Sedimentary
4) Zero Hour
5) Privilege Motion
108. On which one of the following issues can SEBI
penalize any company in India ?

35
A) Violation of Banking Regulation Act.
B) Violation of foreign portfolio investment
guidelines.

111. Expand the term ALM as used in Banking/


Finance sector ?
1) Asset Liability Mismatch

C) For violation of Negotiable instrument Act.

2) Asset Liability Maturity

1) Only (A)

3) Asset Liability Management

2) All (A), (B) and (C)

4) Asset Liability Manpower

3) Only (A) and (B)

5) None of these

4) Only (B) and (C)


5) Only (B)
109. In the opinion of the governor of reserve Bank
of India, which one of the following is the
reason owing to which Indias inflation is
accelerating ?
1) Excess liquidity in market
2) Speculation in essential goods
3) Higher food cost
4) Commodities futures
5) None of these
110. In opinion of the RBI, which one of the
following is/are a threat to the smooth
functioning of Indian Companies ?

112. What are teaser loan rates charged by banks ?


1) Fixed rate of interest charged by banks
2) Floating rate of interest charged by banks
3) Rate of interest in the initial period is less
and goes up subsequently
4) Rate of interest in the initial period is more
and it goes down subsequently
5) None of these
113. What is the full form of the term LIBOR as
used in financial/banking sectors?
1) Local Indian Bank Offered Rate
2) London-India Bureau of regulations
3) Liberal International Bank Official ratio

A) Rising input costs.

4) London inter Bank Offered Rate

B) High interest rates.

5) None of these

C) Inflation only.
1) Only (A)
2) Only (B)
3) Only (A) and (B)
4) Only (C)
5) All (A), (B) and (C)

114. Expand the term FSDC which is used in


financial sectors ?
1) Financial Security and Development Council
2) Financial Stability and development council
3) Fiscal Security and Development Council
4) Fiscal Stability and Development Council

36
5) None of these
115. One single statement that depicts the financial
position of a bank and/or business enterprise at
a given point of time is called_____
1) Statement of product details
2) Reconciliation statement

4) Mutual
5) Ministry
119. One of the major challenges banking industry is
facing these days is deliberate efforts of some
people to bring money earned through illegal
activities in circulation. Which of the following
acts is passed to prevent this activity ?

3) Balance Sheet

1) Payment & settlements Act

4) Quarterly returns submitted to RBI

2) Banking regulation Act.

5) Trading and manufacturing account

3) Negotiable Instrument Act

116. Banks borrow money from the RBI on which of


the following rates ?
1) Reverse Repo Rate
2) Repo Rate

4) Narcotics and Psychotropic Substance Act


5) Prevention of Money laundering Act
120. Which of the following organizations provides
guarantee to the exporters ?

3) SLR

1) Exim Bank

4) CRR

2) Export Credit Guarantee Corporation

5) Savings Rate

3) Director General Foreign trade

117. The Reverse mortgage scheme is launched to


give benefit to which of the following groups of
society ?
1) Govt. employees
2) Senior Citizens
3) Unemployed youth
4) War widows
5) None of these
118. What does the letter M depict in the term
SME as used in the financial world ?
1) Maximum
2) Medium
3) Mercantile

4) reserve Bank of India


5) Registrar of companies
121. Which of the following terms is not used in
Banking World ?
1) Credit
2) Rate
3) Financial Status
4) Discount
5) Absolute Zero
122. What is the full form of IRR as used in
banking/financial sectors?
1) Internal Rate of Return

37
2) Internal Revaluation Reserve
3) Investment Reserve Ratio
4) Internal Risk Return
5) None of the above
123. Which of the following organizations provides
credit history of the borrowers?

5) None of these
126. Banking and financial services, all over the
world, are regulated usually by monetary
Authority of the land. Who controls this
function in India?
1) Ministry of Finance
2) SEBI

1) CIBIL

3) RBI

2) ARCIL

4) IRDA

3) SEBI

5) FEDAI

4) RBI
5) CCIL
124. Banks need liquidity to meet which of the
following objectives of banking?
A) Meet deposit withdrawal
B) Fund loan demands
C) Maintain public confidence
1) Only (A)
2) Only (B)
3) Only (C)
4) Only (A) and (B)
5) Only (B) and (C)
125. Banks make frequent changes in their product
profile including introduction of new products
etc. This is called__
1) Product Control
2) Product enhancement
3) Product marketing
4) Product enrichment

127. As per guidelines of the RBI banks are to


provide appropriate banking facilities to
habitations having population in excess of 2000
by which date?
1) 2011
2) 2012
3) 2015
4) 2016
5) None of these
128. FIMMDA stands for--1) Foreign Investment Markets & Derivatives
market Association
2) Fixed Income Money Markets & Derivatives
Association
3) Fixed Income &
Development Association

Money

Market

4) Floating Income
Derivative Assets

Money

Markets

&

5) None of these
129. Which of the following terms is NOT used in
banking?

38
1) Debit Card

3) RBI

2) Credit Card

4) SBI

3) Kisan card

5) NABARD

4) ELISA-Test
5) None of these
130. Which of the following is/ are the objective(s)
of our monetary policy?

133. In which one of the following States, Indias


first Islamic bank is proposed to be set up?
1) Kerala
2) Tamil Nadu

A) Anchor inflation expectations

3) Orissa

B) Actively manage liquidity

4) Bihar

C) Maintain interest rate regime consistent with


price output and financial stability
1) Only (A)

5) None of these
134. Which of the following terms is NOT used in
Economics?

2) All (A), (B) and (C)

1) Demand and Supply

3) Both (A) and (C)

2) Ad Valoram Tax

4) Only (B)

3) Break even

5) None of these

4) HIV Positive

131. For Which of the following purposes) RBI has


constituted a Working group with Shri Deepak
Mohanty as Chairman?
1) Deregulation of primary markets
2) Deregulation of secondary markets
3) Deregulation of Savings Bank Rates
4) Introduction of Base Rate
5) None of these
132. Which of the following will set up core banking
infrastructure for rural banks?
1) SIDBI
2) IBA

5) Cost benefit analysis


135. For which one of the following reasons Finance
Minister denied entry of NRI funds for infra bonds?
1) it will bring in more liquidity in the country
2) NRIs can withdraw their funds any time
3) It will bring interest burden on the country
4) It could lead to losses at the time of
redemption because of fluctuations in the
Indian Currency
5) None of these
136. What is financial inclusion?
A) Easy access to bank accounts for safe
parking savings

39
B) Availability of cheap credits through
appropriately designed loans for poor and low
income households and small entrepreneurs

C) CRR/SLR

C) Availability of basic financial products like


insurance

2) Only (B)

1) Only (A)

4) Only (C)

2) Only (B)

5) Both (A) and (C)

3) Only (C)
4) Both (A) and (B)
5) All (A), (B) and (C)
137. Which of the following terms is used in
Economics?

1) Only (A)

3) All (A),(B) and (C)

140. Which one of the following rates signal the


RBIs long term outlook on interest rates?
1) Repo Rate
2) Reverse Repo Rate
3) Bank Rate

1) Keynesian

4) SLR

2) Adsorption

5) CRR

3) Affinity Matrix
4) Gene Flow
5) None of these
138. Which one of the following is the directive
given to the Govt. of India in the Financial
Stability Report submitted to it?

141. What do you understand by Para banking


services?
1) Eligible financial services rendered by banks
2) Utility services provided by banks
3) Services provided
correspondents

through

business

1) Focus on financial consistency

4) Services provided to armed force personnel

2) To reduce fiscal deficit

5) None of these

3) To ensure GDP growth


4) To reduce revenue deficit
5) None of these
139. Which one of the following are key policy rates
used by RBI to influence interest rates?
A) Bank rate/ the Repo rate
B) Reverse Repo Rate

142. For which one of the following reasons,


NABARD has set up joint liability groups (JLG) of
farmers?
1) To facilitate delivery of credit to farmers
2) To facilitate better delivery, of credit to
farmers through in formal sources
3) To facilitate better delivery of credit to share
croppers and those who do not have their own
land

40
4) To facilitate better delivery of credit to
farmers through Panchayats
5) None of these
143. Which of the following terms is NOT used in
Banking World?
1) Holding Company
2) Post dated cheque

4) Both (A) and (B)


5) All (A), (B) and (C)
146. As per the decision taken by the Government of
India two financial inclusion funds are set up
with an overall corpus of Rs. 500 crore each.
These funds Financial Inclusion Fund and
financial Inclusion technology Fund are to be
set up by the initial contribution made by the

3) Credit

1) SIDBI, NABARD and UTI

4) Time deposit

2) NABARD, UTI and RBI

5) Centripetal force

3) Government of India, NABARD and RBI

144. Which of the following is the Central bank of


U.S.A.?
1) Federation of Banks in U.S.A.
2) Citigroup U.S.A.
3) Bank of America

4) SIDBI, Government of India & IDBI Bank


5) None of these
147. Banks these days have launched a new scheme
product known as Reverse Mortgage. The
scheme is designed keeping which of the
following groups of society in consideration?

4) Central Bank of U.S.A.

1) Youngsters who have just started earning

5) Federal Reserve System

2) Defence personnel whose life is always at


high risk

145. Which of the following is the correct definition


of the term commercial papers?
A) It is nothing but the popular name of the
Judicial stamp papers used to register financial
transactions.
B) It is one of the instruments through which
Corporates raise debt from the market.
C) It is the name of the Certificate of
Deposits provided by the Banks to its retail
customers.
1) Only(A)
2) Only (B)
3) Only (C)

3) Senior Citizen
4) Women who do not have in dependent
source of income
5) None of these
148. Many a times we read in the newspapers a term
HOT MONEY Which of the following is the
correct definition of HOT MONEY?
A) This is the fund which is dumped into a
country to get the advantage of a favourable
interest rate and hence brings higher returns.
B) This is the fund which is provided by a bank
in USS at very short notice and at a very

41
high rate of interest and for a longer period
of repayment.
C) This is the fund which is pushed into market
through Hawala or some other such illegal
methods and some times referred also as
Black Money.
1) Only (A) is correct
2) Both (A) and (B) are correct
3) Only (C) is correct
4) Both (A) and (C) are correct
5) Only (B) is correct
149. The real return to the investor sometimes gets
reduced due to sudden rise in the prices of the
commodities. This phenomenon in financial
market is known as correct

1) Only (A)
2) Only (B)
3) Only (C)
4) All (A) and (C)
5) None of these
151. The Sensitive Index of the Bombay Stock
exchange keeps on changing positively and
negatively both. Which of the following is/are
the main reasons which contribute for his
sudden change in the same?
A) Higher GDP Growth
B) Continuous Foreign Funds Flow
C) High Volume of Forex reserves
D) Recession in USA

1) Market risk

1) Only (A) and (B)

2) Inflation risk

2) Only (B) and (C)

3) Credit risk

3) Only (C) and (D)

4) Diversification of funds

4) Only (B)

5) None of these

5) All (A), (B) (C) and (D)

150. As per the reports in the leading newspapers


Securities and Exchange Board of India (SEBI)
has asked the Mutual Fund industry to stop
Misselling their schemes to investors. What
is Misselling of products?
A) Misselling takes place when mutual funds
are sold without telling the likely returns.
B) When agents sell the products without
telling investors what are the risks involved
in investing in mutual funds.
C) When agents invest somebodys money in
mutual funds without their knowledge, it is
called misselling.

152. As per the reports published in the newspapers


the banks, particularly public sector banks are
tying up with various rating agencies for
providing a qualitative assessment of the credit
needs of the borrowers. Which amongst the
following is/are such credit rating agencies in
India?
A) CARE
B) CRISIL
C) ULIP
1) Only A
2) Only B

42
3) A and B only

1) Credit cards

4) Only C

2) Loan against gold from financial institute

5) All A, B and C

3) Debit cards

153. As per the newspaper reports the RBI is


planning to introduce plastic Currency Notes.
A) Their Shelf life will be longer.
B) It will plastic money or credit debit cards
which are giving birth to many fraudulent
practices.

4) Money lender
5) None of these
156. Base-II norms are associated with which of the
following aspects of the banking industry?
1) Risk management

C) Printing will be cheaper

2) Manpower planning

1) Only C

3) Retirement benefits for the employees

2) Only B

4) Corporate Governance

3) Only A

5) None of these

4) All A, B and C
5) None of these
154. Sub Prime Lending is term applied to the
loans made to

157. What is meant by Underwriting the term


frequently used in financial sector?
1) Under valuation of the assets
2) The act of taking on a risk for a fee

1) those borrowers who do not have a good


credit history.

3) giving a Guarantee that a loan will not


become a bad loan

2) those who wish to take loan against the


mortgage of tangible assets

4) The act of permission to float an IPO

3) those who have a good credit history and are


known to bank since 10 years
4) those borrowers who are most preferred
customers to the Bank
5) None of these
155. As per the reports published in various journals
and newspapers the small borrowers in rural
areas still prefer to take informal route for their
credit needs. Which of the following is the
informal route of credit in financial sector?

5) None of these
158. As we have noticed many banks of Indian
origin are opening offices/branches in foreign
countries. Why is this trend emerging at a very
fast pace?
A) These Banks wish to provide as banking
facilities to foreigners as banking facilities
are not plenty in many foreign countries.
India wants to take an advantage of the
situation.
B) These banks wish to help Indian firms to
acquire funds at internationally competitive rates.

43
C) These banks wish to promote trade and
investment between India and other countries.
1) Only A

40 Exposure limit
5) Diffusion
161. Which of the following is NOT one of the
recommendations of the committee setup on
Financial
sector
Reforms
under
the
Chairmanship of Raghuram G. Rajan.

2) Only B
3) Only C
4) All A, B and C

1) Give more freedom to banks to setup


branches and ATM any where

5) Only B and C
159. Many economists, bankers and researchers in
India often advocate that banks should equip
themselves for new challenges. These
challenges are in which of the following
shapes/ forms?
A) As Indian economy is getting increasingly
integrated with the rest of the world the
demand of the corporate banking is likely
to change in terms of size, composition of
services and also the quality.
B) The growing foreign trade in India will have
to finances by the local banks.
C) Foreigners are habitual of the comforts
provided by the technology, India has to do
a lot in this reference.
1) Only A is correct

2) set up an office of the financial ombudsmen.


3) All deposits taking institutions should be free
from the supervision of the RBI.
1) Only A
2) Only B
3) Only C
4) All A, B and C
5) Only B and C
162. The Reserve bank of India (RBI) keeps on
modifying various rates/ratios to keep the flow
of liquidity in the market in a balanced
situation.
Which
of
the
following
rates/ratios/indexes is NOT directly controlled
by the RBI?

2) Only B is correct

1) Cash reserve Ratio (CRR)

3) Only C is correct

2) Bank Rate

4) All A, B and C are correct

3) Repo Rate

5) None of these

4) Reserve Repo Rate

160. Which of the following


banking/finance related term?
1) Credit wrap
20 EMI
3) Held to Maturity

is

NOT

5) Wholesale Price Index (WPI)


163. Many banks have adopted/launched Core
banking Solution is_
1) a marketing strategy adopted by the banks.

44
2) a new type of ATM useful for rural
population.

167. Banking Sector will fall under which of the


following sectors?

3) a delivery channel for quick and fast delivery

1) Agricultural sector

4) a new product launched to help senior


citizens only as they are able to visit
branches/ATMs frequently.

2) Service sector

5) None of these

4) Industrial sector

164. Which of the following cannot be called as a


Value Added Service offered by a bank?
1) Special Accounts for poor sections of
society.

3) Manufacturing

5) None of these
168. An account in which trading of shares in their
electronic form is done, is known as___
1) Demat Account

2) Accident Insurance Cover

2) NRI Account

3) Instant Credit of outstation cheques

3) NRIO Account

4) Free cheque books.

4) Current Account

5) All are value added services.

5) None of these

165. Opening the Savings bank Account of a minor


girl will be called as which of the following in
Banking terminology.
1) Retail banking
2) Merchant Banking
3) Institutional banking
4) Social Banking
5) Corporate Banking
166. Which of the following terms is NOT used in
Banking Sector?
1) SLR
2) NPA
3) Credit rating
4) Fixed
5) PURA

169. Which of the following is NOT a measure of


the risk Management in Banks?
1) CRR
2) RTGS
3) SLR
4) Deposit Insurance
5) All are the measures of risk management
170. RBIs open market operation transactions are
carried out with a view to regulate_______
1) Liquidity in the economy
2) Prices of essenties commodities
3) Inflation
4) Borrowing power of the banks
5) All of the above

45
171. When more than one banks are allowing credit
facilities to one party in coordination with each
other under a formal arrangement the
arrangement is generally known as_______
1) Participation

3) An instrument in the form of depository


receipt created by an Indian depository against
underlying equity shares of the issuing
company
4) An instrument in the form of deposit receipt
issued by Indian depositories

2) Consortium

5) None of these

3) Syndication

175. Reverse repo means_______

4) Multiple banking

1) Injecting liquidity by the Central bank of a


country through purchase of govt. securities.

5) None of these
172. Open market operations, one of the measures
taken by RBI in order to control credit
expansion in to economy means______

2) Absorption of liquidity from the market by


sale of Govt. securities.
3) Balancing liquidity with a view to enhance
economic growth rate

1) Safe or purchase of Govt. securities


2) Issuance of different types of bonds

4) Improving the position of availability of the


securities in the market

3) Auction of gold
4) to make available direct finance to borrowers

5) Any of the above


176. The stance of RBI monetary policy is________

5) None of these

1) inflation control with adequate liquidity for


growth

173. The bank rate means____


1) Rate of interest charged by commercial
banks from borrowers
2) Rate of interest at which commercial banks
discounted bills of their borrowers.
3) Rate of interest allowed by commercial
banks on their deposits
4) Rate at which RBI purchases or rediscounts
bills of exchange of commercial banks.

2) improving credit quality of the banks


3) strengthening credit delivery mechanism
4) supporting investment demand in the
economy
5) Any of the above
177. What is Call Money?
1) Money borrowed or lent for a day or over
night

5) None of these
174. What is an Indian depository Receipt?
1) A deposit account with a Public sector bank

2) Money borrowed for more than one day but


upto 3 days

2) A depository account
depositories in India

3) Money borrowed for more than one day but


upto 7 days

with

any

of

46
4) Money borrowed for more than one day but
up to 14 days
5) None of these
178. The branding line of Bank of Baroda is_____
1) International Bank of India
2) Indias International Bank
3) Indias Multinational bank
4) Worlds local Bank
5) None of these
179. The logo of Bank of Baroda is known as____
1) Sun of Bank of Baroda
2) Baroda Sun
3) bank of Barodas Rays
4) Sunlight of bank of Baroda
5) None of these
180. One of the major challenges banking industry is
facing these days is money laundering. Which
of the following , acts. Norms are launched by
the banks to prevent money laundering in
general?
1) know Your customer Norms
2) Banking regulation Act
3) Negotiable Instrument Act
4) Narcotics and Psychotropic Substance Act
5) None of these
181. Lot of Banks in India these days are offering
M-Banking Facility to their customers. What is
the full form of M in M-Banking?
1) Money

2) Marginal
3) Message
4) Mutual Fund
5) Mobile Phone
182. Which of the following is/are true about the
Sub-Prime Crisis? ( The term was very much
in news recently).
a) It is a mortgage crisis referring to Credit
default by the borrowers.
b) Sub-Prime borrowers were those borrowers
who were rated low and were high risk borrowers.
c) This crisis originated because of negligence
in credit rating of the borrowers.
1) Only A
2) Only B
3) Only C
4) All A, B and C
5) None of these
183. Which of the following is NOT the part of the
scheduled banking structure in India?
1) Money Lenders
2) Public Sector Banks
3) Private Sector Banks
4) Regional rural Banks
5) State Co-operative Banks
184. Very often we read in newspapers/magazines
about Sovereign Wealth Funds. Which of the
following is/are the correct description of the
same?
A) These are the funds or the reserves of a
government or central bank of a country

47
which are invested further to earn profitable
returns.
B)

These are the funds, which were


accumulated by some people over the years
but were not put in active circulation as they
retain them as Black Money for several
years.

C) The funds which are created to be used as


relief funds or bailouts packages are known
as sovereign funds.
1) Both (B) and (C)

C) Change in government revenue and


expenditure to influence the level of national
output and prices.
1) Only A
2) Both A and B
3) Only C
4) All A, B and C
5) None of these

2) Only (B)

187. Which of the following services is introduced


recently by the reserve Bank of India and SEBI
in India for the first time?

3) Only (C)

A) Trading of Currency futures.

4) Only (A)
5) None of these
185. The Ratio of the Cash reserves that the banks
are required to keep with RBI is known as_
1) Liquidity Ratio
2) SLR
3) CRR
4) Net Demand and Time Liability
5) None of these
186. Very often we read about the Monetary Policy
of the RBI in a newspapers/magazines. What
is/are the objectives of the Monetary Policy of
the RBI?

B) Opening of NRI Accounts in Indian


currencies.
C)

Foreign Direct Investment through


Participatory Notes (PNs) by the Foreign
Institutional Investors.

1) Only A
2). Only B
3) Only C
4) All A, B and C
5) None of these
188. The rate at which the RBI gives loans to the
commercial banks is called
1) Repo rate

A) to maintain price stability and ensure


adequate flow of credit to the productive sector
of the economy.

2) Bank rate

B) Stability for the national currency, growth in


employment and income are also looked into.

4) CRR

3) Reverse Repo

5) None of these

48
189. Which of the following is NOT the part of the
organized sector of India Money Market?
1) Mutual Funds

4) Provide financial service to the people of the


nation of its origin across the country
5) Sell the goods at subsidized cost

2) Non-Banking financial Companies

193. What is the Repo Rate?

3) Unit trust of India

1) It is a rate at which RBI sell Government


securities to banks

4) Reserve Bank of India


5) Chit funds
190. Which of the following cannot be called as a
Debt instrument as referred in financial transactions?
1) Certificate of Deposits
2) Bonds
3) Stocks
4) Commercial of papers
5) Loans
191. Whenever RBI does some Open Market
Operation transactions, actually it wishes to
regulate which of the following?
1) Inflation Only

2) It is a rate at which RBI buys government


securities from banks
3) It is a rate at which RBI allows small loans
in the market
4) It is a rate which is offered by banks to their
most valued customers or prime customers.
5) None of these
194. Which of the following correctly describes
what sub-prime lending is?
A) Lending to the people who cannot return the
loans.
B) Lending to the people who are high value
customers of the banks.

2) Liquidity in economy

C) Lending to those who are not a regular


customer of a bank.

3) Borrowing powers of the banks

1) Only A

4) Flow of foreign Direct Investments

2) Only B

5) None of these

3) Only C

192. In economics it is generally believed that the


main objective of a Public Sector financial
company like Bank is to__
1) Employ more and more people
2) Maximize total profits
3) Maximize total production

4) All, A, B and C
5) None of these
195. Which of the following is/are the major
concepts visible in todays banking industry in
India?
A) Risk Based Management
B) Growing competition

49
C) IT Initiatives
1) Only A

manual. This step of the organization can be


classified under which one of the following
categories of measures for a business?

2) Only B

1) Preventive

3) Only C

2) Compliance

4) Both B and C

3) Corrective

5) All A, B and C

4) Detective

196. Which of the following products of a bank is


specifically designed to provide financial help
to children in their higher studies in India or in
a foreign nation?

5) None of these
199. A customer wishes to purchase some US dollars
in India. He/She should go to_

1) Personal Loan

1) Public Debt Division of the RBI only

2) Corporate Loan

2) American Express Bank only

3) Educational Loan

3) RBI or any branch of a bank which is


authorized for such business

4) Mortgage Loan
5) None of these
197. Which of the following is the limitation of the
ATMs owing to which people are required to
visit branches of the bank?
A) It does not accept deposits.
B) It has a limited cash disbursement capacity.
C) Lack of human interface.
1) Only A
2) Only B
3) Only C
4) All A, B and C
5) None of these are limitations
198. A Bank/ Financial Organization these days
relies heavily on e-commerce for its
transaction. As a part of system security, it has
introduced organizations security awareness

4) Ministry of Foreign Affairs


5) None of these
200. Very often we see in the advertisements
published by Financing Institutes/Agencies
stating that their products are given high or
average Ratings. These Rating Agencies
classify Bonds/investments in how many
categories?
A) Low Risk
B) Average Risk
C) High Risk
1) Only (A)
2) Only (B)
3) Only (C)
4) All (A),(B) and (C)
5) None of these

50
201. Banks and other institutions have issued debit
and credit cards the purpose of both are:

204. The Negotiable Instruments Act defines the


holder in due course as

1) the same, to make paperless payments

1) Holder of a negotiable instrument.

2) Different, since in credit card, the account is


credited with the amount while in debit card
the account is debited.

2) A person who is entitled to hold before it


becomes due for payment

3) the same, there is risk weightage of 125 % in


both the cards
4) different, since in debit cards interest for
delayed period is charged while in credit
cards no such interest is charged by banks.
5) None of these
202. When does an automatic right of set-off arise?
1) on death, insanity or insolvency of the
customer
2) on receipt of garnishee order
3) on receipt of assignment of credit balance
4) on the happening of an event as in (1), (2) or
(3)
5) None of these
203. Can the cheque-returning memo be treated as
evidence in offences under Negotiable
Instruments Act?
1) It is secondary evidence, banker could be
called in court

3) A person who is possessor of the Negotiable


instrument for consideration
4) All of the above
5) None of these
205. As per the provisions of section 138 of
Negotiable Instruments Act, 1881 payee of a
cheque may initiate criminal action. If the
cheque is returned for the reason
1) Refer to drawer
2) Payment stopped by the drawer
3) Post-dated
4) Insufficient funds
5) None of these
206. Partners are mutual agents. It applies to
sleeping or secret partners also. The rights of other
partners_
1) can be restricted and entrusted to one partner
2) cannot be restricted and not be entrusted to
one partner

2) it is prima facie evidence. Banker need not


be called in court

3) can be enjoyed jointly

3) It is affidavit evidence, banker to tender


affidavit

5) None of these

4) Both (1) and (2)


5) None of these

4) can be enjoyed severally

207. When Co-operative banks are functioning in


more than one State, then they are governed by
1) Multi State Co-operative Societies Act, 1984
2) Multi Units Co-operative Societies Act

51
3) Multi State Co-operative Societies Act,2002
4) Banking Laws (Application to Co-operative
Societies) Act, 1965
5) None of these
208. Banking Regulation Act, 1949 deals with
1) The regulation of banking companies
2) The control over the management of banking
companies
3) Suspension and winding up of banking
companies
4) All of the above
5) None of these
209. Under what circumstances can the bank close a
partnership account?
1) Death of Partner
2) Retirement of a partner
3) Insolvency of a partner
4) All of the above
5) None of these
210. Power of attorney is a stamped document and
generally executed in the presence of
1) A Notary Public
2) A Magistrate of a Court
3) An Authorised Government official
4) All of the above
5) None of these
211. Can banks lend below their PLR? If so, to
which sectors?

1) No because the PLR is the minimum


lending rate for banks
2) No- because due to competition, and the
high cost of funds, the banks cannot afford to
lend below PLR
3) Yes banks can lend below PLR to special
category of borrowers I, e. exporters, public
enterprises, agriculture, education loans and
women entrepreneurs.
4) Yes banks can lend below PLR for advance
below Rs. 2 lakhs.
5) None of these
212. Winding up of a banking company is done
under the provisions of the Banking Regulation
Act, 1949. The reasons for such winding up can
be
1) Inability to pay its debts.
2) Moratorium placed on the bank/failure to
comply with minimum capital adequacy.
3) Failure to comply with requirements of
Banking regulation Act/disentitled to carry
on banking business/compromise or
arrangement by the court cannot be
worked/continuance is prejudicial to the
interests of the depositors.
4) All of the above
5) None of these
213. The advantages of Local Area Net work to
banks are:
1) Sharing of expensive resources by several
users.
2) Availability of stored information to all
users.
3) All the terminals are intelligent terminals;
the processing load is shared by computers in LAN.

52
4) All of the above

3) SBI

5) None of these

4) SIDBI

214. Which is the card that looks like any other


plastic card or an ATM card with integrated
circuit (IC-Chip)?

5) None of these
217. Disadvantages to Credit Card holders include:

1) Member card

1) Over Spending ending in Debt Trab

2) Charge card

2) Frauds due to loss or theft of cards

3) Credit card

3) Forged signatures

4) Smart card

4) All of the above

5) None of these

5) None of these

215. Which one is true in respect of bank marketing?


1) Bank marketing deals with providing
services to satisfy customers financial needs
and wants.

218. The nature of charge created on the intangible


securities like accounts receivable is________
1) Assignment
2) Mortgage

2)
Bank
marketing
has
to
discover/ascertain/anticipate the financial
needs of the corporate.

3) Lien

3) Bank marketing may be required to satisfy


the corporates, and institutions other related
needs and wants.

5) None of these

4) Bank marketing means competitive


element, efficiency and effectiveness in the
process.
5) None of these
216. National financial Switch has been set up to
facilitate connectivity between the Banks
switches and their ATMs, and interbank
payment gateway for authentication and routing
the payment details of various e-commerce
transactions. Which organisation has set up this
switch?
1) IBRD
2) IDRBT

4) Only (1) and (2)

219. The possession of the mortgaged property is


normally given to the mortgagee in the case of____
1) Simple mortgage
2) English Mortgage
3) Usufructuary Mortgage
4) All of the above
5) None of these
220. RBI has been playing its role of supervision,
control and development of the monetary and
banking system in the country through its
following functions. Which is not true?
1) Public Debt Management

53
2) Banker to Government
3) BankersBank
4) Banker to the general public
5) All are correct
221. The instruments used by RBI under general
credit control are:
1) Bank Rate
2) CRR

224. State which one amongst the following is not a


tool of Monetary?
1) CRR
2) SLR
3) Branch Licensing Policy
4) Open Market Operations
5) None of these
225. The percentage of SLR currently in force is

3) SLR

1) 25

4) Exchange Control

2) 24

5) Only (1) , (2) and (3)

3) 23

222.________ is a tool for general credit control


used by RBI which not only influences the flow
of liquidity for the purpose of expansion or
contraction of credit but also helps RBI to
stabilize money supply and prices of
government securities:
1) Bank Rate
2) CRR/SLR
3) OMO
4) Direct Auction
5) None of these
223. Exchange control methods adopted by RBI
include:

4) 22
5) None of these
226. The demutualised exchanges in India do not
include:
1) Bombay Stock Exchange
2) National Stock Exchange
3) OTCEI
4) Ahmedabad Stock Exchange
5) None of these
227. The Mutual Funds in India are regulated by:
1) RBI

1) Bilateral settlement of mutual claims

2) SEBI

2) Import restrictions and tariffs

3) AMFI

3) Export subsidies and changes in interest


rates

4) IRDA

4) Changes in Bank rate


5) Only (1). (2) and (3)

5) None of these
228. The Basel II Accord is based on the concept of
a capital ratio where the numerator represents

54
the required amount of capital by a bank and
the denominator:
1) The risk-weighted assets
2) Non-performing assets
3) Only (1) and (2)
4) All of the above
5) None of these
229. The Basel II accord differs from the earlier
accord by the introduction of an explicit
treatment for_______ in the definition of riskweighted assets:
1) Credit Risk
2) Operational Risk

If one bill purchased/discounted by a bank is


overdue for more than 90 days. The
outstanding____ is treated as NPA.
1) Of the overdue bill.
2) In the account towards all the bills
purchased/discounted.
3) In the borrowers various advances accounts
with the Bank.
4) All of the above
5) None of these
232. A loan granted for short term crops will be
treated as NPA. If the instalment of principal
or interest thereon or both remain overdue
for______ crop season(s).

3) Market Risk

1) one

4) All of the above

2) two

5) None of these

3) three

230. An account should be treated as out of order


if the outstanding balance remains continuously
in excess of the sanctioned limit/drawing
power. In cases where the outstanding balance
is less than the sanctioned limit/drawing power,
but there are no credits continuously for__ days
or the credits are not enough to cover the
interest during the same period, then the
account should be treated as out of order
1) 90 days
2) 120 days
3) 180 days
4) 200 days
5) None of these
231. The bill remains overdue for a period of 90
days. In the case of bills purchased and discounted.

4) four
5) None of these
233. ABC Bank has advanced Rs. 50,000 to a farmer
for cultivation of sugarcane crop in June 2010.
The borrower has repaid Rs. 25,000 plus
interest accrued on the entire loan promptly
after the harvest of sugarcane in June, 2011 and
informed the bank that he could not pay the
balance amount of loan as the crop was affected
by the disease resulting in a poor yield just
sufficient to pay a half of the loan availed by
him from the bank. How would the bank
recognise the balance loan outstanding in his
account for the purpose of asset classification
as on 30th June, 2011?
1) As a Standard Asset
2) As a Sub-Standard Asset

55
3) As a Doubtful Asset

1) 20%

4) As a Loss Asset

2) 30%

5) None of these

3) 50%

234. The sub-standard accounts subjected to


restructuring would be eligible to be upgraded
to the standard category only after a specified
period of___ after the date on which the first
payment of interest or principal, whichever is
earlier, falls due:

4) 100%
5) None of these
237. By financial inclusion, we mean provision of
___________ by the format financial system to
those who are hitherto excluded.

1) 90 days

1. Affordable financial Services

2) 180 days

2. Savings and insurance Services alone

3) One year

3. Payments and remittance facilities alone

4) Two years

4. Small value credit facilities

5) None of these

5. None of these

235. In cases where the loan is sanctioned by the


bank against the borrower submitting the fake/
forged documents and thus playing fraud on the
bank, such a loan is to be treated in the banks
books as a_____ asset, even when there is
recovery in the account as per the repayment
schedule approved by the bank at the time of
sanctioning the loan.

238. The RBI, in its efforts towards faster financial


inclusion both in the rural and urban areas, has
also advised banks to simplify the KYC
procedure for opening accounts for those
persons with balances not exceeding _ and
credits____ in a year.
1) Rs. 1000 and Rs. 10000

1) Standard

2) Rs. 5000 and Rs. 25000

2) Sub-standard

3) Rs. 25000 and Rs. 50000

3) Doubtful

4) Rs. 50000 and Rs. 1 lakh

4) Loss

5) None of these

5) None of these
236. Banks shall make 100% provision against the
unsecured portion of a doubtful asset. In regard
to the secured portion and if the asset remained
in doubtful category for less than a year, a
provision of ______ may be made on the
amount of advance so secured:

239. Under the Business Facilitator Model, banks


may use intermediaries such as:
1) NGOs/Farmers Clubs
2) Post Offices
3) KVIC/KVIB Units
4) All of the above

56
5) None of these
240. The scope of the activities undertaken by the
business correspondents will include:
1) Disbursal of small-value credit
2) Recovery of loans and interest
3) Collection of small-value deposit

2) 2 months
3) 3 months
4) 4 months
5) None of these
244. Primary Dealers (PDs) deal in _______ in both
primary and secondary markets.

4) All of the above

1) Equities

5) None of these

2) Debentures

241. Banks cannot use the services of the following


persons as Business facilitator/Business
Correspondent:
1) Village Sarpanchs
2) Insurance Agents

3) Corporate Bonds
4) Government Securities
5) None of these
245. Urban Co-operative banks are controlled by

3) Ex-servicemen

1) State governments and RBI.

4) Retired government Officials

2) State Governments and NABARD

5) None of these

3) Both State and Central Governments

242. When banks make advances to limited


companies against their assets, the required
forms are to be presented to the registrar of
Companies within_________ from the date of
execution.

4) RBI and NABARD


5) None of these
246. Among the following, which one is not a
regulator?

1) 21 days

1) RBI

2) 30 days

2) SEBI

3) 1 month

3) AMFI

4) 2 months

4) IRDA

5) None of these

5) None of these

243. In the case of creation of registered mortgages,


the mortgage deed should be presented for
registration with registrar of Assurances within
___ from the date of execution of the deed:
1) 1 month

247. RBI is popularly known as a lender_____


1) To Central Government
2) To State Government

57
3) Of foreign Currency
4) Of the last resort
5) None of these
248. What is a financial system?
1) Financial system means a mechanism in an
economy to mobilise the monetary
resources/capital from various surplus
sectors of the economy and to allocate and
distribute the same to various needy sectors.
2) A system to access the demand and supply of
the finance.
3) An arrangement to finance the farmers of the
country.
4) A system to put financial management on the
borrowers of the schedule commercial banks
of the country.
5) None of these
249. Central Monetary Authority of India is:

251. Which of the following is authorised to act as


agent of Reserve Bank of India at the places
where there is no branch of Reserve Bank of
India?
1) Central bank of India.
2) State Bank of India.
3) All banks situated at that place.
4) All Nationalised Banks
5) None of these
252. Banking is defined wide Section 5 (1) (b) of the
Banking regulation Act, 1949 as :
1) The accepting, for the purpose of lending or
investment, of deposits of money from the
public, repayable on demand or otherwise
and withdrawable by cheque, drafts, order or
otherwise
2) Accepting deposits and making advances to
public.

2) Reserve Bank of India.

3) Accepting money and keeping the same in


their vault and repay the same on the
demand of customer.

3) Planning commission

4) All of the above

4) Central Bank of India.

5) None of these

1) Government of India, Ministry of Finance.

5) None of these
250. Which of the following banks is not wholly
owned by the government of India?

253. RBI was established in 1935 pursuant to the


recommendations of:
1) The Hilton Young Commission.

1) Reserve Bank of India.

2) All India Rural credit Survey Committee.

2) State Bank of India.

3) Gorawala Committee.

3) Punjab and Sind Bank.

4) Talwar Committee.

4) Central bank of India.

5) None of these

5) None of these.

254. RBI was nationalized in :

58
1) 1935
2) 1949
3) 1955
4) 1969
5) 1947
255. On 19th July 1969, 14 banks were nationalised
these banks had deposits of more than:
1) Rs. 25 crore.
2) Rs. 75 crore
3) Rs. 85.crore

258. As per section 6 of the Banking regulation Act,


a banking company can undertake:
1) Merchant banking
2) issue of guarantee and indemnity.
3) Executor and trustee business
4) All of the above
5) None of these
259. RBI has now permitted banks to carry on the
business of Leasing, Hire Purchase and
factoring directly by themselves, the leadings
under each category should not exceed:

4) Rs. 100 crore.

1) 1% of the total bank credit.

5) None of these.

2) 5% of the total bank credit

256. The first public sector bank to issue capital to


public is :
1) Corporation Bank.
2) Indian Overseas Bank
3) Oriental Bank of Commerce.
4) Punjab National Bank
5) None of these
257. The Banking Companies Act, 1949 was enacted
to consolidate and amend the law relating to
banking companies with effect from 1st March
1966; the name of the act has been changed as :

3) 10% of the total bank credit.


4) All of the above
5) None of these
260. Under Section 20 of Banking Regulation Act, a
bank is prohibited from granting any loan or
advance to :
1) Any of its directors.
2) Any firm (s) in which directors are interested
as partner, manager etc.
3) Any company in which a director is
interested as Managing Director etc.

1) Negotiable Instruments Act.

4) All of the above

2) Reserve Bank of India Act.

5) None of these

3) The Banking and Transfer of Undertaking


Act.
4) The Banking Regulation Act.
5) None of these

261. Section 17 of the Banking regulation Act lays


down that banks shall create a reserve fund out
of the balance of profit of each year, also before
the declaration of dividend a certain percentage
of such profit should be transferred to the
reserve fund. This percentage is :

59
1) 10% of such profit.

4) Talwar Committee

2) 20% of such profit

5) None of these

3) 15% of such profit.


4) 25% of such profit.
5) None of these
262. A scheduled bank is one, the name of which is
included in the second scheduled of RBI Act,
1934. Such a bank should have a paid up
Capital and reserves of an aggregate value of
not less than :
1) Rs. One lakh
2) Rs. Five lakh
3) Rs. Ten lakh

265. The working of RRBs was reviewed in 1986


and their continuance and greater involvement
of the sponsor bank in their management was
recommended by
1) Narsimham Committee.
2) Kelkar Committee
3) Khusro committee
4) Marathe committee
5) All of the above
266. The regulatory Authority for Regional Rural
Banks is:

4) Rs. Hundred lakh

1) RBI and NABARD.

5) None of these

2) Sponsoring Bank.

263. The Land Development Banks grant medium


term loan for Land Development etc. against
the mortgage of lands. In some states these
banks are known as :
1) Primary Agricultural Credit Societies
2) State Co-operative Banks
3) Urban Co-operative Banks
4) Land Mortgage Banks
5) None of these
264. In 1992 RBI had appointed a committee to
make recommendations to tone up the working
of the Urban Co-operative Banks. The name of
the committee was:

3) State government and Central government


and Sponsoring Bank in the ratio of 15 : 50 :
35.
4) All of the above
5) None of these
267. RRBs have been making contribution in the are
of rural credit since f reception still they suffer
from the following weaknesses:
1) Poor capital structure.
2) Increasing overheads and decreasing profits.
3) Lack of effective coordination between
Commercial Banks. Co-operative Banks and
RRBs in the area of their operation

1) Marathe committee

4) All of the above.

2) Narsimham Committee

5) None of these

3) Tondon committee

60
268. Regional Rural Banks are empowered to
transact the business of banking as defined under:

2) undertake both mobilisation of resources as


well as proper development among various sectors.

1) Section 5(b) of Banking Regulation Act,


1949.

3) carry out the role of intermediaries between


the money market and Cen-tral Co-operative Banks.

2) Union Budget every year.

4) All of the above.

3) Regional Rural Banks Act 1976.

5) None of these

4) Negotiable Instruments Act, 1881.


5) All of the above
269. The working funds of primary agricultural
societies is obtained from :
1) Owned funds comprising share capital,
entrance fees and the reserve fund.
2) Deposits from members and non-members.
3) Loans from Co-operative Central Banks and
the governments.
4) All of the above.
5) None of these
270. Central Co-operative Banks:

272. The first Land Mortgage Bank was established


in :
1) 1920 at Jhind in Punjab.
2) 1929 at Chennai
3) 1947 at New Delhi.
4) 1962 at Jammu
5) None of these
273. Land Mortgage Bank provides long term credit
against the security of :
1) Crop
2) Mortgage of land generally for a period of 5
to 20 years.

1) serve as the connecting links between State


Co-operative Banks and Primary Credit
Societies.

3) Intangible assets.

2) finance the Primary Credit Societies, balance


the excess and deficiency in their resources
but do little commercial banking

5) None of these

3) are closer to the Primary Societies than an


Apex Bank
4) All of the above
5) None of these
271. The state Co-operative Banks:
1) assume the key position in the co-operative
credit structure.

4) Only (1) and (2)

274. The main objective of Land Development Bank


is :
1) Providing investment credit for agriculture.
2) Providing crop loans to the farmers.
3) Developing allied activities in rural and
semi-urban areas.
4) Creating good social relations in the
residents of rural and semi-urban areas.
5) None of these

61
275. The regulatory authority for the activities of
Merchant Banker in India is :
1) Reserve Bank of India.
2) Ministry of Finance.
3) Securities and Exchange Board of Delhi.
4) Indian Banks Association.
5) All of the above.
276. What are major functions undertaken by
merchant bankers?

4) All of the above


5) None of these
279. What are the major activities of NABARD?
1) it is provides refinance to State Co-operative
Banks Scheduled commercial Banks and
Regional Rural Banks etc
2) It co-ordinates the activities of different
agencies in the field of agriculture and rural credit.
3) It improves absorptive capacity of credit
delivery system

1) Issue Management.

4) All of the above

2) Capital structuring/restructuring.

5) None of these

3) Market maker in capital market.


4) All of the above
5) None of these
277. Which of the following is not the function of a
merchant banker?
1) Collection of bills and cheques
2) Issue Management
3) Preparation of project reports
4) Syndication of loans

280. Automatic refinance Scheme is available to the


persons financed under:
1) the scheme of setting up of Agriclinic and
Agribusiness centres.
2) Rural non-farm sector (investment credit)
upto Rs. 15 lakh
3) Composite Loan Scheme.
4) all of the above.
5) None of these

278. The followings constitute the code of conduct


for merchant bankers as stipulated by SEBI :

281. For loans and advances granted by commercial


banks to small road and water transport
operations scheme (SRWTED), NABARD
provides refinance under Automatic refinance
facility subject to the conditions:

1) Maintenance of high degree of standards of


integrity and fairness in dealings

1) Repayment of loan should not exceed 5


years with moratorium of 6 months.

2) Providing true and adequate information to


investors and abide by the provisions of
various acts, rules and regulations etc.

2) Refinance amount is restricted to Rs. 15 lakh


per borrower.

3) Ethical conduct of business and provision of


information to customers in respect of code.

3) The borrower should be from rural area and


he should utilise the vehicle mainly for
transportation of rural farm and non farm

5) All of the above

62
products and inputs and passengers to/from
marketing centres.
4) All of the above
5) None of these
282. NABARDs refinance is available
to
commercial banks under nonfarm sector for
investment activities of :
1) Artisans
2) Small scale industries, tiny sector, village
and cottage industries.

4) Joint stock companies and co-operative


societies engaged in the manufacturing and
processing of goods.
5) None of these
285. Which is an apex body in the field of industrial
finance in the country?
1) Industrial development Bank of India.
2) EXIM Bank.
3) Industrial credit and Investment corporation
of India Ltd.

3) Handicrafts, handlooms, powerlooms etc.

4) ECGC

4) All of the above

5) None of these

5) None of these
283. The Industrial Finance Corporation of India is
the first institution established to build a term
financing institutional structure in India; it was
established in :

286. The small Industries Development Bank of


India (SIDBI) was set up by the Government of
India in April 1990 as a wholly owned
subsidiary of :
1) RBI

1) July 1948

2) IFCI

2) August 1969

3) IDBI

3) September 1957

4) ICICI

4) December 1970

5) None of these

5) None of these

287. The objectives of setting up SIDBI are:

284. The Industrial Finance corporation of India


provides loans to :

1) To initiate steps for technological up


gradation and modernisation of existing units.

1) government companies only which are in


core sector ) i.e./ power generation, steel, coal
and cement etc.),

2) To expand channels for marketing of SSI


sector products in India and abroad.

2) Newly established industries in backward


districts.

3) To promote employment-oriented industries


in semi-urban areas and to check migration of
population to big cities.

3) Industries in private sector.

4) All of the above


5) None of these

63
288. The main functions of SIDBI are :
1) to provide refinance for loans and advances
extended by the primary lending institutions
to SSI units and also to provide resources
support to them.
2) It discounts and rediscounts bill arising from
sale of mechinery to or manufactured by
industrial Units in the SSI sector.
3) It extends seed capital/soft loan assistance
under National Equity Fund, Mahila Udyam
Nidhi and Mahila Vikas Nidhi and Seed
Capital Schemes through specified lending
agencies.
4) All of the above
5) None of these
289. The discount and Finance House of India
(DFHI) was established in April 1988 as per the
recommendations of :
1) Working group on Money Market headed by
Shri N.Vaghul.
2) Narsimham Committee on Banking reforms.
3) Reserve bank of India.
4) Only (2) and (3)
5) All of the above
290. Export-Import Bank of India was set up as a
statutory corporation, wholly owned by
government of India, in :
1) January 1981.
2) August 1947.
3) January 1984
4) January 1965
5) August 1965

291. Export-Import Bank of India has been set up


for the purposes of :
1) functioning as a specialized institution for
providing credits to foreign trade on
international competitive terms.
2) offering advisory services to exporters.
3) providing refinance facilities in regard to
export finance extended by commercial
banks and other financial institutions.
4) All of the above
5) None of these
292. The following are the schemes of assistance
extended by Export-Import Bank of India :
1) It extends as assistance to exporters engaged
in export of plant, equipment, machinery
and related services by way of medium term
loans for the period exceeding six months.
This credit assistance enables exporters to
extend deferred credit to the foreign buyer.
2) It extends financial assistance to Indian
promoters who promotes overseas joint
ventures to support their equity investments
through overseas investment finance.
3) It extends overseas buyers credit to foreign
importers for import of Indian Capital goods
and related services.
4) All of the above
5) None of these
293. Which is the correct statement in respect of
Exim Banks Lending Programme for Export
Oriented Units?
1) Interest rate on rupee term loan is linked to
Banks minimum lending rate.

64
2) Interest rate on foreign currency term loan
may be floating of fixed based on banks cost of
funds.

3) To provide professional management of


portfolio, to help the small investors to earn
the relatively higher rate of return.

3) Interest rate is charged half yearly.

4) All of the above

4) All of the above.

5) None of these

5) None of these
294. Under its foreign Currency Pre-shipment credit
(FCPC), Exim Bank gives short term foreign
currency finance to eligible exports for
procurement of inputs and to commercial banks
for on-lending to export customers; the terms of
finance are:
1) Interest rate on finance is maximum 2% over
LIBOR (London Inter Bank Offered Rate).
2) Maximum period of finance is 180 days
from the date of disbursement.
3) For commercial banks, loans availed from
EXIM Bank are exempted from the
requirement of Cash Reserve Ratio,
Statutory Liquidity requirement and
Incremental
Credit-deposit
ratio
requirements.
4) All of the above.
5) None of these
295. Unit trust of India (UTI) was set up under an
Act of Parliament in 1964, or Statutory Public
Sector Investment Institution. The objectives of
the institutions are:
1) To provide to the investors or small and
moderate means the same advantages of
investments as enjoyed by the large
capitalists.
2) To diminish or minimise the risk of
investment in stocks by spreading or
diversifying investments over a large
number of different kinds of stocks.

296. National Housing Bank was established under


National Housing Bank Act, 1987 and started
functioning from 9th July 1988, on the
recommendations made by a High Level Group
Headed by:
1) Dr. C. Rangarajan
2) Dr. Bimal Jalan
3) Mr. Narasimham
4) Mr. I . G Patel
5) None of these
297. The mam objectives of National Housing Bank
are :
1) To promote and develop specialised housing
finance institutions for mobilisation of
resources and extending finance for housing
activities.
2) To extend refinance facilities to housing
finance institutions and to scheduled banks.
3) To provide guarantee and underwriting
facilities to housing finance institutions.
4) All of the above
5) None of these
298. For availment of refinance from National
Housing Bank, Scheduled Commercial Bank
has to satisfy the following conditions:
1) The capital adequacy ratio of the banks
should be as per the norms prescribed by the
reserve Bank of India.

65
2) The Net Non-Performing Assets to the net
advances of the Bank should not exceed
10%

1) NBFCs falling short of the stipulated


minimum Net Owned Funds (NOF) were
precluded from accepting public deposits.

3) The Scheduled Commercial Bank has earned


profit for the last two years.

2) An NBFC not having minimum credit rating


as prescribed by RBI is not eligible to accpt
fresh deposits.

4) All of the above.


5) None of these
299. A company is deemed as Non-banking
Financial Company :
1) If its financial assets are more than 50% of
its total tangible assets.
2) If its income from financial assets is more
than 50% of the gross income of the company.
3) Both (1) and (2)
4) Financial assets are more than 40% of its
total assets.
5) None of these
300. As per the provisions of the Reserve Bank of
India (Amendment) Act, 1997, Non-Banking
Finance Company has :
1) to apply to the Reserve Bank of India for a
certificate of registration.
2) to have a minimum net owned fund (N.O.F.)
of Rs. 15 lakh

3) Ceiling on the quantum public deposits was


related to the level of credit rating given by
the approved credit rating agencies.
4) All of the above
5) None of these
302. The word Bank is derived from:
1) German word back; which means a joint
stock fund.
2) Italian word banco
3) Words bancus; or banque which means a
bench
4) All of the above
5) None of these
303. The following bank was one of the presidency
banks :
1) The Bank of Bengal
2) Bank of Bombay

3) Both (1) and (2)

3) Bank of Madras

4) Minimum NOF of Rs. 10 lakh

4) All of the above

5) None of these

5) None of these

301. In January 1998, the reserve Bank of India


introduced new regulatory framework for
safeguarding the interest of depositors. The
guidelines comprises :

304. The three presidency banks were merged in


1921 with the establishment of:
1) State Bank of India
2) Imperial Bank of India
3) Punjab National Bank

66
4) Reserve Bank of India

1) Checking bank failures

5) None of these

2) ensuring adequate branch expansion in rural


areas

305. The report of All India Rural Credit Survey


Committee (Gorawala Committee) led to the
formation of

3) Consolidating and amending the law relation


to banking companies.

1) Reserve Bank of India

4) All of the above

2) State Bank of India

5) None of these

3) The Imperial Bank of India


4) Bank of Bengal
5) None of these
306. Which Bank had taken over the Imperial Bank
of India?
1) State Bank of India
2) Reserve Bank of India
3) Indian Bank

309. A customer has a right to ask the bank to return


him a paid instrument before the expiry of
period of such instruments under the provisions
of :
1) Section 131 of the Negotiable Instruments
act, 1881.
2) Section 34 of Reserve Bank of India Act,
1934.
3) Section 45 Z of the Banking regulation Act,
1949.

4) Central Bank of India

4) All of the above

5) None of these

5) None of these

307. The Banking Companies Act, 1949 was enacted


to consolidate and amend the law relating to
banking companies, with effect from 1 st March
1966, the name of the Act has been changed
as :
1) The Banking regulation Act, 1949.
2) The Banking and Transfer of Undertaking
Act.

310. Section 14 A (1) of the banking regulation Act,


1949 prohibits a banking company from :
1) allowing advance against its own shares.
2) creating floating charge on the undertaking
or any property of the company or any part thereof
3) disclosing secret reserves created by it.
4) All of the above

3) Negotiable Instrument Act.


4) D.I.R. Act.
5) None of these
308. The Banking Regulation Act, 1949 was enacted
with the objective of:-

5) None of these
311. Section 26 of the Banking Regulation Act, 1949
requires every banking company to submit an
annual return to the Reserve Bank of India, in
respect of :
1) advances to agriculture sector.

67
2) all accounts in India which have not been
operated upon for ten years.
3) advances allowed to priority sector.

raised to Rs. 300 crore within three years of


commencement of business:

4) details of its demand and time liabilities

2) minimum capital adequacy ratio should be at


10 percent on continuous basis from the
commencement of business.

5) None of these

3) Both (1) and (2)

312. As per the provision of Section 29 of the


Banking regulation Act, 1949 every banking
company is required to prepare its final
accounts; viz., Profit & Loss account and
Balance Sheet in the firms prescribed in :
1) the Second Schedule to the RBI Act, 1934
2) the Third schedule to the Banking regulation
Act, 1949.

4) No limit of capital
5) None of these
315. The main thrust of the branch licensing policy
is :
1) to provide freedom to banks to rationalise
the structure of their branches.

3) Table A to the Companies Act, 1956.

2) to give an opportunity to the banks to open


as many branches as they wish.

4) All of the above

3) Both (1) and (2).

5) None of these

4) to increase number of bank branches in the


country

313. As per guidelines of reserve Bank of India, a


new private sector bank :
1) shall be subject to prudential norms in regard
to income recognition, asset classification
and provisioning, capital adequacy etc.
2) shall have to observe priority sector lending
targets as applicable to other domestic
banks.
3) will be required to open rural and semi-urban
branches.

5) None of these
316.
Narasimham
recommendations include:

committees

main

1) adoption of uniform accounting practices


particularly in respect of income recognition
and provisioning against doubtful debts.
2) minimum 4% capital adequacy ratio to risk
weighted assets by March 1993.

4) all of the above

3) imparting transparency to bank balance


sheets and full disclosures in them.

5) None of these

4) All of the above.

314. As per guidelines issued by RBI on 3 rd January,


2001:
1) the minimum paid up capital requirement of
a newly set up private sector bank should be
at least Rs. 200 crore and the same should be

5) None of these
317. The Banks are required to constitute high
power liability management committees
comprising top executives to determine suitable
policies for:

68
1) each liability product such as demand
deposits, savings, term deposits and CODs.
2) liabilities denominated in foreign exchange.
3) monitoring ratio yield elastic and inelastic
liabilities in the total liabilities as also
supply statement of use of call money
borrowings.
4) All of the above
5) None of these
318. The world over most of the supervisory
authorities have adopted the following as the
basis of assessment of capital adequacy :
1) Risk assets ratio System.
2) Return or capital System.
3) Average Yield on Assets System.

4) All of the above.


5) None of these
321. Tier-II capital consists of :
1) Undisclosed reserves
perpetual preference shares.

3) subordinated debt, hybrid debt capital


instruments.
4) All of the above.
5) None of these
322. As per RBI stipulation, the total of Tier II
elements will be limited to a maximum of _ %
of total of Tier I elements for the purpose of
compliance with the banks.
1) 50%

5) None of these

2) 100%

1) Net capital
2) Gross capital.
3) Working capital.
4) Core capital.
5) None of these.
320. Tier-I capital would mean:
1) Paid up capital.
2) Statutory reserves and other disclosed free
reserves.
3) Capital reserves representing surplus arising
out of sale proceeds of assets.

cumulative

2) general provisions, loss reserves and


revaluation reserves.

4) Net profit.

319. The Basic committee has defined capital in two


tiers (Tier-I and Tier-II). Tier-I capital is kinown as :

and

3) 200%
4) 75%
5) 82%
323. What do you mean by the money market?
1) A place where money is sold and bought.
2) It is nothing but a place where two
currencies are exchanged.
3) Money market is a market for short terms
financial assets (generally upto the tenure of
12 months) that are close substitutes of
money
4) All of the above
5) None of these

69
324. Who are the major players in the call money
markets?
1) Banks.
2) Insurance companies.
3) Development Financial Institutions
4) Selected Mutual funds
5) All of the above
325. Treasury Bill means:
1) a bill drawn on a treasury.
2) an instrument is used by the Central
government for shorterm borrowing.
3) a bill drawn by any authority by sitting in
treasury office.
4) a bill drawn by Member of Parliament.
5) All of the above
326. Who issues treasury bills?
1) Reserve Bank of India, as the agent of the
Central government
2) Any Government Department of State and
Central.

4) Commercial Bills
5) All of the above
328. Certificate of deposit refers to :
1) a certificate issued by reserve bank of India,
stating the amount of deposit held by a bank.
2) a certificate issued by regional Rural Bank of
its depositors.
3) a negotiable money market instrument,
issued in dematerialised form or as a
issuance promissory note, for funds
deposited at a bank or other eligible
financial institution for a specific time
period.
4) All of the above
5) None of these
329. How much is the minimum amount for issuance
of certificate of deposit?
1) It should not be less than Rs. 1 lakh and in
multiples of Rs. 1 lakh thereafter.
2) Rupees 1 crore and in multiples of Rs. 10
lakh, thereafter.
3) Rupees 10 lakh.

3) Any Commercial Bank.

4) No minimum stipulated answer

4) All of the above

5) None of these

5) None of these
327. Which are the various instruments in the Indian
Money Market?
1) Call money and short notice money
instruments.
2) Inter bank term deposits/ loans.
3) Treasury Bills

330. What is the minimum period before which


Certificate of Deposit is not transferable ?
1) No such minimum period. Certificate of
Deposit can be transferred at any time before its
maturity
2) 15 days from the date of issue
3) Three months from the date of issue
4)15 days prior to maturity

70
5) 30 days from the date of issue
331. A commercial paper refers to :
1) an unsecured money market instrument
issued in the form of promis-sory note.
2) a paper issued by commercial organisations.
3) a secured money market instrument
4) a letter issued by scheduled commercial
banks for seeking refinance from NABARD.
5) None of these
332. Commercial paper can be issued by :
1) Corporates
2) All India financial Institutions that have been
permitted to raise short term resources under
the umbrella limit fixed by RBI
3) Primary dealers
4) All of the above
5) None of these
333. Commercial paper is :
1) fully secured instrument.
2) unserved money market instrument.
30 partly secured paper.
4) All of the above
5) None of these
334. Who can make investment in a commercial
paper?
1) Individuals.
2) Banking companies.
3) Corporate bodies registered or incorporated
in India and unincorporated bodies.

4) None resident Indians (NRIs) and foreign


Institutional Investor (Flls).
5) All of the above.
335. The Commercial banking System in India
comprises of
1) Scheduled and Non scheduled Banks.
2) Nationalised Banks and Private Sector
Banks.
3) Regional Rural Banks and Co-operative
Banks.
4) Land Development Banks
5) None of these
336. Which of the following were considered to be
compelling reasons for Bank Nationalisation?
1) Concentration of wealth and economic
power in the hands of industrialists and
businessmen.
2) Branch expansion was confined to urban
areas and rural areas were being neglected.
3)

Sectors like agriculture, small scale


industries and the other deserving sectors
were outside the purview of bank lending
operations.

4) Various malpractices indulged in by banks


under private ownership.
5) all of the above.
337. Enumerate the main functions of a banker as
per Banking regulation act, 1949:
1) accepting deposit of money
2) advancing of loans
3) Issuance of letters of credit
4) all of the above

71
5) None of these
338. The Banking regulation act, 1949, applies to the
following categories of banks:
1) Nationalised Banks.
2) Non-nationalised Banks
3) Co-operative Banks.
4) All of the above
5) None of these
339. A Bank incorporated in India cannot open an
office outside India without obtaining prior approval
of

2) details of deposits and advances


3) advances to weaker sections of the society
4) dividend payment made
5) None of these
342. Under the provisions of section 35 (b) (ii) of the
Banking regulation act, 1949, inspection of
branches of Indian banks situated abroad is to
be carried out by:
1) Reserve Bank of India
2) Exim Bank.
3) Ministry of External affairs

1) Government of India.

4) All of the above

2) Reserve Bank of India

5) None of these

3) Exim Bank.
4) Joint Director of Foreign Trade
5) None of these
340. The provision of section 21 of the Banking
regulation act, 1949, empowers reserve Bank of
India to :
1) inspect the banking companies.
2) to regulate CRR and SLR.
3) control advances by banking companies.
4) All of the above
5) None of these
341. Section 26 of the Banking regulation Act, 1949,
requires every banking company to submit an
annual return to the reserve Bank of India, in
respect of :
1) all accounts in India which have not been
operated upon for ten years and above

343. Section 29 of the Banking regulation act, 1949,


stipulates that every banking company has to
prepare its final accounts i.e., Profit and Loss
Account and Balance Sheet, in the forms
prescribed in
1) the third schedule to the Banking Regulation
act 1949.
2) the second schedule of the reserve Bank to
India Act, 1934.
3) the second schedule to the companies Act,
1956.
4) the third schedule to the Negotiable
Instruments Act , 1881.
5) None of these
344. The term ways and Means advances refer to:
1) the temporary advance made to the
government by its bankers to bridge the internal
between expenditure and the flow of receipt of
revenue.

72
2) the advance given by the banks to the
poorest of the society.
3) lending made under PMRY scheme
4) All of the above
5) none of these
345. In terms of section 19 of the RBI Act, 1934, the
RBI has been prohibited from:
1) making loans or advances
2) drawing or accepting bills payable otherwise
than on demand
3) allowing interest on deposits or current
accounts
4) All of the above
5) None of these
346. Cash reserve ratio is maintained in the form of :

348. Statutory Liquidity Ratio means:


1) The maximum percentage of demand and
time liabilities upto which bank can extend
the advances
2) Maintenance of liquid assets by banks in the
form of cash, gold and unencumbered
approved securities equal to not less than
25% of their total demand and time deposit
liabilities under the provisions of Section 24
of Banking regulation Act.
3) To keep liquid cash.
4) All of the above
5) None of these
349. The reduction in the Statutory Liquidity ratio
by the reserve Bank of India:
1) will improve the resources of banks.

1) Government securities.

2) will augment the resources of scheduled


commercial banks.

2) Balance with Reserve Bank of India

3) will not affect the liquidity position.

3) Balance with State Bank of India.

4) All of the above

4) All of the above.

5) None of these

5) None of the above.


347. RBI is empowered to prescribe cash Reserve
Ratio ranging between :
1) five percent to twenty percent of net demand
and time liabiliaties.
2) three percent to fifteen percent of net
demand and time liabilities.
3) three percent to forty percent of net demand
and time liabilities
4) Any one of (1), (2) and (3)
5) None of these

350. Under the powers derived from Sections 20, 21


and 21A of the Reserve Bank of India Act, 1934:
1) the Reserve Bank manages the public debt
and issues new loans on behalf of the
Central and State Governments.
2) the Reserve Bank provides refinance to
NABARD in respect of agricultural
advances.
3) the Reserve Bank regulates flow of credit to
the economy by variations in the statutory
Liquidity Requirement and Cash Reserve
Ratio.

73
4) All of the above

23. (2) Priority Sector

5) None of these

24. (3) All (A), (B) and (C)

ANSWERS

25. (3) Balance between price of all essential


commodities

1. (4) All (1), (2) and (3) above


2. (5) National Bank for Agriculture and Rural
development (NABARD)

26. (4) All (A),(B) and(C)


27. (2) useful but need strict regulation

3. 5) Germany

28. (5) All (A), (B) and (C)

4. (1) IRDA

29. (4) All (A),(B) and (C)

5. (2) Double Fault

30. (4) 1%

6. (4) Double Dip recession

31. (5) Wobbe Index

7. (1) Loss Data Consortium

32. (4) Banks should prepare a road map for


financial inclusion covering rural households

8. (4) Only (a) and (b)

33. (4) International Financial Reporting Standards

9. (5) Liquidity Adjustment Facility

34. (2) It is a mechanism to protect firms exposure


to market risk

10. (2) Only (a)


11. (1) Only (A)

35. (1) The currency is backed by Government


Guarantee

12. (2) Net Demand & Time Liabilities


13. (3) ALL (A), (B) and (C)
14. (3) Bank
Committee

for

International

Settlements

15. (5) (A) and (C)


16. (2) April 2012
17. (5) Reducing taxes and increasing subsidies
18. (3) Banks to Banks
19. (3) Minimum Alternate Tax
20. (3) Society for World wide Interbank Financial
Telecommunication

36. (2) is a legal contract-between the owner of an


asset and its user, containing many conditions
including payment of a specified amount by the
user to the Owner for use of the asset.
37. (2) is selling of account receivables on a contract
basis for cash payment to a factor before it is due.
38. (4) RBI and Banking Regulation Act.
39. (4) It discourages certain kinds of lending by
assigning higher risk weights to loans it deems
undesirable
40. (5) Home Loans
41. (3) IIFCL

21. (1) Future contract

42. (1) LIC

22. (2) Ministry of Commerce

43. (2) Increase in credit growth

74
44. (4) It results into deflation
45. (4) RBI and banking Regulation Act
46. (5) Planning Commission
47. (3) Fluctuates with market price movements
48. (1) Amounts are deposited and are withdrawn as
per requirement of the customers.

68. (1) To ensure that Companies take steps for


rehabilitation of project affected people
69. (4) Only (1) and (2) above
70. (4) UNIDO
71. (1) ICICI Bank
72. (2) Bangladesh

49. (3) Shares in a demat account

73. (5) Unit Linked Insurance Plan

50. (1) Salaried class

74. (3) Help to solve balance of payment problems


of member countries.

51. (2) RBI


52. (5) RBI
53. (3) Governor, Reserve Bank of India
54. (2) Pledge
55. (1) is used to rate the borrowers while giving
advances

75. (1) Plan for financial Inclusion


76. (1) Money Lenders
77. (2) RBI
78. (1) Core
79. (5) All (1) (2) and (3)

56. (1) Liquidity risk

80. (1) April 2011

57. (3) ICICI Bank

81. (1) Repo Rate

58. (3) International Financial Reporting standards

82. (1) Only A

59. (2) 12 days

83. (3) Only A

60. (4) Real Total Gross Securities

84. (1) those borrowers who do not have a good


credit history.

61. (1) meet its current liabilities


62. (1) initial price offered by a private limited
company to its share holders

85. (4) Money lender


86. (1) Risk management

63. (3) a strained liquidity

87. (1) Under valuation of the assets

64. (5) the existing profitability of the company.

88. (5) Only B and C

65. (5) a, b, d, e

89. (5) Diffusion

66. (3) All (A) (B) and (C)

90. (1) Only A

67. (5) Gulbarga

91. (3) It is buying of one currency and selling of


another currency

75
92. (1) Financial Literacy and Credit Counselling

115. (3) Balance Sheet

93. (3) It is a gap between total expenditure and total


receipts of the Government

116. (2) Repo Rate

94. (3) Administrative Ministries only


95. (5) Central Rural Sanitation programme
96. (1) Bangladesh
97. (3) Business correspondent

117. (2) Senior Citizens


118. (2) Medium
119. (5) Prevention of Money laundering Act.
120. (2) Export Credit Guarantee Corporation
121. (5) Absolute zero

98. (2) ASBA : Application Supported by Blocked


Amount.

122. (1) Internal Rate of Return

99. (1) Only (A) and (B)

123. (1) CIBIL

100. (4) Bank Rate

124. (4) Only (A) and (B)

101. (4) wealth management

125. (3) Product marketing

102. (3) RBI

126. (3) RBI

103. (2) All (A), (B) and (C)

127. (2) 2012

104. (4) It is to protect NBFCs from any impact of


possible economic slowdown

128. (2) Fixed Income Money Markets and


Derivatives Association

105. (1) Risk management

129. (4) ELISA-Test

106. (2) NABARD

130. (2) All (A), (B) and (C)

107. (1) Interest Rate Swap


108. (5) Only B

131. (5) Shri Deepak Mohanty Committee was


constituted to make the balance of payments
manual for India.

109. (3) Higher food cost

132. NABARD

110. (5) All (A), (B) and (C)

133. (1) Kerala

111. (3) Asset Liability Management

134. (4) HIV positive

112. (3) Rate of interest in the initial period is less


and goes up subsequently

135. (2) NRIs can withdraw their funds any time

113. (4) London Inter Bank offered Rate


114. (2) Financial Stability and Development
Council

136. (1) Only (A)


137. (1) Keynesian
138. (2) To reduce fiscal deficit
139. (3) All (A), (B) and (C)

76
140. (3) The bank rate signals the central bank long
term outlook on interest rates. If the bank rate
moves up, long term interest rates also tend to
move up and viceversa.

156. (1) Risk management

141. (2) Para Banking is a kind of banking wherein


money is accepted for the purpose of saving
from an individual as in case of a normal
banking function. The acceptance of money
under Para banking is scheduled daily,
monthly, quarterly, half yearly, yearly and
even for fixed period for more than one year.

159. (4) All A, B and C are correct

The Only major difference between a normal


banking and Para Banking is that, under Para
Banking one cannot option for current account
facility and carry its day to day transaction for
accepting and withdrawl for funds. Also, a
depositor cant issue any cheques against the
amount in its Para banking saving schemes.
142. (1) To facilitate delivery of credit to farmers
143. (5) Centripetal force
144. (5) Federal Reserve System
145. (2) Only (B)
146. (3) Government of India, NABARD
147. (3) Senior Citizen
148. (1) Only (A) is correct
149. (1) Market risk
150. (4) All (A), (B) and (C)
151. (5) All (A), (B) (C) and (D)
152. (3) A and B only
153. (3) Only A
154. (2) those who wish to take loan against the
mortgage of tangible assets
155. (4) Money lender

157. (2) The Act of taking on a risk for a fee


158. (1) Only A

160. (5) Diffusion


161. (1) Only A
162. (5) Wholesale Price Index (WPI)
163. (3) a delivery channel for quick and fast
delivery
164. (4) Free cheque books.
165. (1) Retail Banking
166. (5) PURA
167. (2) Service sector
168. (1) Demat Account
169. (2) RTGS
170. (5) All of the above
171. (2) Consortium
172. (2) Issuance of different types of bonds
173. (4) Rate at which RBI purchases or rediscounts
bills of exchange of commercial banks.
174. (3) An instrument in the form of depository
receipt created by an Indian depository against
underlying equity shares of the issuing
company
175. (1) Injecting liquidity by the Central bank of
country through purchase of Govt. securities.
176. (1) inflation control with adequate liquidity for
growth
177. (1) Money borrowed or lent for a day or over
night

77
178. (2) Indias International Bank
179. (2) Baroda Sun
180. (1) Know Your Customer Norms
181. (5) Mobile Phone
182. (4) All A, B and C
183. (1) Money Lenders

203. (2) It is Prima facie evidence, banker need not


be called in court
204. (4) All of the above
205. (4) Insufficient funds
206. (1) can be restricted and entrusted to one
partner

184. (3) Only (C)

207. (3) Multi State Co-operative Societies Act,


2002

185. (2) SLR

208. (4) All of the above

186. (2) Both A and B

209. (4) All of the above

187. (1) Only A

210. (4) All of the above

188. (1) Repo Rate

211. (3) Yes-banks can lend below PLR to special


category of borrowers i.e. exporters public
enterprises, agriculture, education loans and
women entrepreneurs.

189. (5) Chit funds


190. (5) Loans
191. (3) Borrowing powers of the banks
192. (2) Maximize total profits
193. (1) It is a rate at which RBI sell Govt. securities
to banks

212. (4) All of the above


213. (4) All of the above
214. (4) Smart Card

194. (1) Only A

215. (1) Bank marketing deals with providing


services to satisfy customers financial needs and
wants.

195. (5) All A, B and C

216. (2) IDRBT

196. (3) Educational Loan

217. (4) All of the above

197. (4) All A, B and C

218. (1) Assignment

198. (1) Preventive

219. (3) Usufructuary Mortgage

199. (3) RBI or any branch of a bank which is


authorized for such business

220. (4) banker to the general public

200. (4) All (A), (B) and (C)


201. (1) the same, to make paperless payments
202. (4) on the happenings of an event as in (1), (2)
or (3)

221. (4) Exchange Control


222. (3) OMO
223. (5) Only (1), (2) and (3)
224. (3) Branch Licensing Policy

78
225. (2) 24
226. (4) Ahmedabad Stock Exchange
227. (2) SEBI
228. (1) The risk- weighted assets
229. (2) Operational Risk
230. (1) 90 days
231. (3) In the borrowers various advances accounts
with the Bank.
232. (2) two
233. (1) As a Standard Asset
234. (3) One year
235. (4) Loss
236. (1) 20%
237. (1) Affordable financial services
238. (4) Rs. 50000 and Rs. 1 lakh
239. (4) All of the above
240. (4) All of the above
241. (1) Insurance Agents
242. (2) 30 days
243. (4) 4 months
244. (4) Governments Securities
245. (1) State Governments and RBI.
246. (3) AMFI
247. (4) of the last resort
248. (1) Financial system means a mechanism in an
economy to mobilise the monetary
resources/capital from various surplus sectors

of the economy and to allocate and distribute


the same to various needy sectors.
249. (2) Reserve Bank of India.
250. (2) State Bank of India
251. (2) State Bank of India.
252. (1) The accepting. For the purpose of lending or
investment, of deposits of money from the
public, repayable on demand or other wise and
withdrawable by cheque drafts, order or
otherwise
253. (1) The Hilton Young Commission.
254. (1) 1935.
255. (2) Rs. 75 crore.
256. (3) Oriental Bank of Commerce.
257. (4) The Banking Regulation Act.
258. (4) All of the above
259. (3) 10% of the total bank credit.
260. (4) All of the above
261. (2) 20% of such profit
262. (2) Rs. Five lakh
263. (4) Land Mortgage Banks.
264. (1) Marathe Committee
265. (2) Kelkar Committee.
266. (1) RBI and NABARD
267. (4) All of the above.
268. (1) Section 5(b) of Banking Regulation Act,
1949.
269. (4) All of the above.
270. (4) All of the above

79
271. (4) All of the above

295. (4) All of the above

272. (1) 1920 at Jhind in Punjab

296. (1) Dr. C. Rangarajan

273. (2) Mortgage of land generally for a period of 5


to 20 years

297. (4) All of the above

274. (1) Providing investment credit for agriculture.


275. (3) Securities and Exchange Board of Delhi.
276. (4) All of the above
277. (1) Collection of bills and cheques
278. (4) All of the above
279. (4) All of the above
280. (4) All of the above
281. (4) All of the above
282. (4) All of the above
283. (1) July 1948
284. (4) Joint stock companies and co-operative
societies engaged in the manufacturing and
processing of goods.
285. (1) Industrial Development Bank of India.
286. (3) IDBI
287. (4) All of the above.
288. (4) All of the above
289. (1) Working Group on Money Market headed
by Shri N. Vaghul.
290. (1) January 1981.
291. (4) All of the above
292. (4) All of the above
293. (4) All of the above
294. (4) All of the above

298. (4) All of the above.


299. (3) Both (1) and (2)
300. (3) Both (1) and (2)
301. (4) All of the above
302. (4) All of the above
303. (4) All of the above
304. (2) Imperial Bank of India
305. (2) State Bank of India
306. (1) State Bank of India
307. (1) The Banking Regulation Act, 1949.
308. (3) consolidating and amending the law relation
to banking companies.
309. (3) Section 45 Z of the Banking Regulation Act,
1949.
310. (2) creating floating charge on the undertaking
or any property of the company or any part thereof
311. (2) all accounts in India which have not been
operated upon for ten years.
312. (20 the Third Schedule to the Banking
Regulation Act, 1949.
313. (4) All of the above
314. (3) Both (1) and (2).
315. (3) Both (1) and (2)
316. (4) All of the above
317. (4) All of the above.

80
318. (1) Risk Assets Ratio System

338. (4) All of the above

319. (4) Core capital

339. (2) Reserve Bank of India

320. (4) All of the above.

340. (3) control advances by banking companies.

321. (4) All of the above

341. (1) all accounts in India which have not been


operated upon for ten years and above

322. (4) 75%


323. (3) Money market is a market for short terms
financial assets (generally upto the tenure of
12months) that are close substitutes of money.
324. (5) All of the above.
325. (2) an instrument is used by the Central
Government for shorter borrowing.
326. (1) Reserve Bank of India, as the agent of
Central Government.
327. (5) All of the above.
328. (3) a negotiable money market instrument,
issued in dematerialised form or a issuance
promissory note, for funds deposited at a bank
or other eligible financial institution for a
specific time period.
329. (4) No minimum stipulated answer.
330. (1) No such minimum period. Certificate of
Deposit can be transferred at any time before
its maturity.
331. (1) an unsecured money market instrument
issued in the form of promissory note.
332. (4) All of the above.
333. (2) unserved money market instrument.
334. (5) All of the above.
335. (1) Scheduled and Non-scheduled Banks.
336. (5) All of the above.
337. (4) All of the above

342. (1) Reserve Bank of India.


343. (1) the third schedule to the Banking
Regulation Act, 1949.
344. (1) the temporary advance made to the
government by its bankers to bridge the
internal between expenditure and the flow of
receipt of revenue.
345. (4) All of the above
346. (2) Balance with Reserve bank of India.
347. (2) three percent to fifteen percent of net
demand and time liabilities.
348. (2) Maintenance of liquid assets by banks in
the form of cash, gold and unencumbered
approved securities equal to not less than 25%
of their total demand and time deposit
liabilities under the provisions of Section 24 of
Banking Regulation Act.
349. (2) will augment the resources of scheduled
commercial banks.
350. (1) the Reserve Bank manages the public debt
and issues new loans on behalf of the Central
and State Governments.

81

Вам также может понравиться